Download Problem solving اعداد فرع الطب الباطني ا د علاء حسين حيدر الحلي

Document related concepts

Patient safety wikipedia , lookup

Dental emergency wikipedia , lookup

Adherence (medicine) wikipedia , lookup

Dysprosody wikipedia , lookup

Management of multiple sclerosis wikipedia , lookup

Multiple sclerosis research wikipedia , lookup

List of medical mnemonics wikipedia , lookup

Transcript
‫‪Problem solving‬‬
‫اعداد فرع الطب الباطني‬
‫ا د عالء حسين حيدر الحلي‬
‫رئيس الفرع‬
‫‪2015-2014‬‬
The main Objectives in these cases :
There are typically four distinct steps objective to the systematic solving
of clinical problems:
1. Making the diagnosis
2. Assessing the severity of the disease (stage)
3. Rendering a treatment based on the stage of the disease 4. Following
the patient’s response to the treatment
Also in the objective of analysis of cases we should :
1. Understand the natural history of the problem.
2. Know the types of differential diagnosis & causes of the problem
3. Be familiar with the complications .
4. Be familiar with indications for management and for prophylactic
medications
5 . Understand the patho physiology of the problem
Problem 1
Diabetic ketoacidosis
Objectives :
1 . To know the common acute complication in diabetes
2 . To learn pathophysiology & proper management
3 . To know the management and complication of this condition
Case history
A 19-year-old man is brought to the emergency department (ED) with
diffuse abdominal pain, vomiting, and altered level of consciousness. The
patient’s symptoms began several days ago, when he complained of acute
URT infection His symptoms at that time included profound fatigue,
nausea, mild abdominal discomfort and some urinary frequency. Today
he was found in bed moaning but otherwise unresponsive. His past
medical history is unremarkable, and he is currently taking no
medications. O/E , the patient appears pale and ill. His temperature is
36.0°C (96.8°F), pulse rate is 140 beats per minute, blood pressure is
80/40 mm Hg, and the respiratory rate is 40 breaths per minute. His head
and neck examination shows dry mucous membranes and sunken eyes;
there is an unusual odor to his breath. The lungs are clear bilaterally with
increased rate and depth of respiration. The cardiac examination reveals
tachycardia, no murmurs ,rubs, or gallops. The abdomen is diffusely
tender to palpation, with hypoactive bowel sounds and involuntary
guarding. The rectal examination is normal. Skin is cool and dry with
decreased turgor. On neurologic examination, the patient
in discomfort and localizes pain but does not speak coherently.
Laboratory studies: pcv 25% the leukocyte count is 16,000 cells/uL, and
the hemoglobin and hematocrit levels are normal. Electrolytes reveal a
sodium of 124 mEq/L, potassium 3.4 mEq/L, chloride 98 mEq/L, and
bicarbonate 6 mEq/L. B.urea and creatinine are mildly elevated. The
serum glucose is 740 mg/dL (41.1 mmol/L). The serum amylase,
bilirubin, AST, ALT, and alkaline phosphatase are within normal limits.
A 12-lead ECG shows sinus tachycardia. His CXR is normal.
1.What's your provisional diagnosis?
2 . Explain the underlying pathophysiological mechanisms for
development of this condition.
3.What are the risk factors which can produce such complication?
4.What are the complications of such condition?
5.What are the outlines of management?
6.How can you explain his low PCV?
7.How can you explain his abdominal pain?
8. After you star your management plan .you reduced his RBS within 1
hour to 200 mg/d. the patient develop headache .confusion. what's your
explanation?
9.How can you correct this new complication?
10.few hours later your patient became well…what are the advises you
would like to give him on discharge?
11 . Explain the underlying pathophysiological mechanisms for
development of this condition.
Problem 2
Stroke
Objectives
1. Recognize the clinical findings of an acute stroke.
2. Understand the diagnostic and therapeutic approach to suspected stroke
patients.
3. Know the most common mechanisms for ischemic stroke: carotid
stenosis, cardio embolism, and small-vessel disease.
4. Understand the evaluation of a stroke patient with the goal of
secondary prevention.
Case History
A 59-year-old man with a history of hypertension presents to the
emergency department(ED)in MTH with right-sided paralysis and
aphasia. The patient’s wife states he was in his normal state of health
until one hour ago, when she heard a thud in the bathroom and walked in
to find him collapsed on the floor. She immediately called emergency
medical services, who transported the patient to your ED. His fingerstick
blood sugar was 108 mg/dL. On arrival in the ED, the patient is placed on
monitors and an IV is established. His temperature is 36.8 oC (98.2°F),
blood pressure is 170/95 mm Hg, heart rate is 86 beats per minute, and
respiratory rate is 20 breaths per minute. The patient has a noticeable leftgaze preference and is verbally unresponsive, although he follows simple
commands such as raising his left thumb. He has a normal neurologic
examination on the left, but on the right has a facial droop, no motor
activity, decreased deep tendon reflexes (DTRs), and no sensation to
light-touch.
1 . What is the most likely diagnosis?
2.What are the risk factors for this condition
3 . How do you explain decrease tendon reflexes?
4 . Is cardiac examination valid and why?
5 . What is the most appropriate next step?
6 . What is the best therapy? Discuss the types of treatments of this patient
7 . What is the anatomical localization of the lesion ? and how could you assess
the prognosis ?
8 . Mention the important factors in the social life of this patient
Problem 3
Childhood Arthritis & its sequel
Case history
A 33-year-old unmarried man was admitted to MTH with a three months history
of increasing oedema . In childhood he suffered a severe and deforming arthritis ,
requiring numerous hospital admissions . For 5 days he had noticed a reduction in
his urinary output . He had been taking prednisolone 7.5 mg daily for the last 10
years .
O/E he looked unwell and was grossly
oedematous with pitting oedema in the ankles , legs & thighs with marked sacral
oedema . His upper limbs were thin and he appeared wasted despite oedema . JVP
not raised BP 120/60 lying and 90/50 standing . His feet were cold . Pulse 80
regular . Dullness at both lung bases . There was wide spread & advanced
deforming arthritis with sublaxation of the joints of both hands . Feet , ankles ,
elbows and knees were also affected , but to lesser extent . Although tender and
painful on movement , the joints were not hot or inflamed . The abdomen
distended with ascitis . The spleen just palpable & liver was 2 cm BCM :
Investigations
Hb 7.9 gm/dl , ESR 100 mm / First Hour, WBC 8.4 X 10ʼ MCV81fl
K 4.3 mmol/L , Na 133mmol/L , HCO3 22 mmol/L, Chloride 95mmol/L
Urea 14mmol/L , Creatinine 80 µmol/L, Ca+2 1.9mmol/L,P 1.1mmol/L
Total Protein 55g/L , albumin 17g/L , LFT Normal
Urine Protein ++++ ; Blood +
1 . What is the differential diagnosis of this problem ?
2 . Consider the important investigations & what important tests that should be
done in regards to proteinuria ?
3 . what treatment you consider in this patient ?
4 . What are the consequences of this condition & their management ?
Problem 4
Inflammatory bowel disease
Objective
1. Understand the initial evaluation and management of IBD.
2. Know the indications for management.
3. Be able to evaluate patients with chronic diarrhea and understand
pathophysiologic mechanisms.
4. Understand the diagnosis, management, and complications of Crohn's
diseases .
Case history
A 34-year-old pharmacologist presented with a history of diarrhea and abdominal
pain . For some years he had noticed some looseness of the bowels , but he
became acutely ill six weeks before presentation , where he was opening his
bowels up to six times a day ; at times the stool contained mucus and occasionally
blood , there is a colicky abdominal pain in the mid abdomen associated with
mild distension . The patient has anorexia and mild weight loss about 4Kg . As
he has mild fever , his GP doctor prescribed a five day course of amoxicillin but
with no benefit . He has no recent travel abroad . O/E the patient appeared pale
and ill , Tempt. 37.4°C , abdomen is diffusely tender , mildly distended , there is
some fullness at the right lower abdomen . PR examination demonstrate perianal
skin tags , painful anal fissure with positive blood . Sigmoidoscopy revealed
hyperaemic rectal mucosa with loss of vascular pattern . Screening test results
Blood picture : Hb 11 g/dl , ESR 66 mm/First Hr, WBC 10000 , MCV 60fl ,
Platelets 520X 10’/ L ,K 3.6 mm/L , Na 140 mm/L , Chloride 101 mmol/L ,
HCO3 28 mmol/L , Blood urea 4.4mmol/L , Creatinine 87ᵤmol /L, Calcium 2.26
mmol/L , serum Phosphate 0.75 mmol/L Total protein 66 g/L , Albumin 28g/ L ,
LFT normal Urine examination is normal , Faecal calprotectin positive
1 . What is the differential diagnosis ?
2 . What further investigations would you consider ?
3 . How could you manage this patient , discussing the role of biologic therapy ?
4 . Mention four renal complications in this patient
5 . In future the patient might need surgical treatment , would you like to mention
the indication in each differential diagnosis .
Problem 5
Acute appendicitis as acute presentation of Crohn's
disease
You are working in the emergency department (ED) of a 15-bed rural
hospital without U/S or CT scan capabilities, and a 25-year-old,
previously healthy, woman presents for evaluation of abdominal pain.
The patient describes her pain as having been present for the past 3 days.
The pain is described as constant, exacerbated by movements, and
associated with subjective fevers and chills. She denies any recent
changes in bowel habits, urinary symptoms, or menses. Her last
menstrual period was 6 days ago. The physical examination reveals
temperature of 38.4°C (101.1°F),pulse rate of 110 beats per minute, blood
pressure of 110/70 mm Hg, and respiratory rate of 18 breaths per minute.
Her skin is nonicteric. Cardiopulmonary examination is unremarkable.
The abdomen is mildly distended and tender in both right and left lower
quadrants. Involuntary guarding and localized rebound tenderness are
noted in the right lower quadrant. The pelvic examination reveals no
cervical discharge; cervical motion tenderness and right adnexal
tenderness are present. The rectal examination reveals no masses or
tenderness.
Laboratory studies reveal:
white blood cell count (WBC) of 14,000 cells/mm3, a normal
hemoglobin, and a normal hematocrit. The urinalysis reveals 3 to 5
WBC/high-power field (HPF), few bacteria, and trace ketones.
1 . What are the most likely diagnoses?
2 . How can you confirm the diagnosis?
3 . You send your patient to a surgeon and the surgeon on laprotomy he
found oedematous small intestine and red oedematous caecum he tried
not do further surgery and he manage to close the abdomen . Why ?
Problem 6
Bleeding esophageal varices
Objectives
1. Learn the differences in presentations and outcomes between upper and
lower GI bleeding.
2. Understand the priorities, evaluations, and management of patients
with GI hemorrhage.
3. Know the causes of UGITB.
4. Learn the complications of chronic hepatitis, such as cirrhosis and
portal Hypertension & ALD .
5. Understand the pathophysiology of oedema & of ascites.
6. Know how to prevent further bleeding .
Case history
You are a house officer working in the emergency unit of MTH when a man
presents at 5 am with moderately sever vomiting blood ( hematemesis ) . At first
glance to the patient you noticed that the patient is confused with impaired mental
status with yellowish skin and sclera . O/E you do indeed noticed some facial
plethora , florid spider angiomata on the face and upper chest , in addition
bilateral Dupuytren's contracture in both hands , fullness at both parotid regions
and gynecomastia , his abdomen is distended and there is bilateral pedal oedema .
BP 95/55 mmHg , PR 110BPM , Tempt 38°C also wasting of the proximal limbs
muscles seen but there is no focal neurological signs . Urgent investigations show
: Hb 11g/dl , WBC count 25X 10’/L , MCV 107fl , B. urea 7.9mmol/L , Bilirubin
420micromol/L , AST 125U/L , ALT 102 U/L , bile pigment +ve in his
urine.Serum K 3.2 mmol/L , Serum Na 132 mmol/L.
1 . What is the first steps in the management of this patient ?
2 . According to clinical presentation of this patient what do you think the
aetiology of his UGIT bleeding & jaundice ? And what further important
investigations you order ?
3 . Why the patient has low serum potassium ?
4 . What is the pathophysiology of his pedal oedema and abdominal distension ?
5 . Give three explanations for high temperature in this patient
6 . Mention how you could prevent further upper GIT bleeding in this
patient in future
Problem 7
CNS infection
Objectives
1. Understand the diagnostic and therapeutic approach to bacterial
meningitis including when to obtain neuroimaging, when to perform a
lumbar puncture, and what empiric therapies to initiate.
2. Recognize the clinical presentation of acute bacterial meningitis.
Case history
An otherwise healthy 19-year-old man is brought to the emergency
department(ED) by his roommate who states that he has “not been acting
right” for the past 24 hours. Per the roommate, the patient had
complained of a headache 2 days prior to arrival, and has been
progressively somnolent and confused since then. The patient has no past
medical history and does not take any medications. His roommate states
that the patient is a college student who does not use any illegal drugs
and. Review of systems is positive for headache and altered mental status
as stated above as well as a tactile fever for the past 2 days. Additional
review of systems is unobtainable as the patient is unable to answer any
questions. On physical examination the patient is noted to be febrile
to 38.5°C (101°F) orally, with a heart rate of 120 beats per minute, blood
pressure of 114/69, and a respiratory rate of 20 breaths per minute. His
oxygen saturation is 98% on room air. The head and neck examination
are significant for dry mucous membranes and nuchal rigidity. His
cardiopulmonary examination is within normal limits with the exception
of tachycardia. The abdomen is soft and nontender. His skin is noted to
be warm and well perfused without any rash. The neurologic examination
is significant for an altered mental status with a Glasgow coma score
(GCS) of 10 (eyes open to voice, patient moans to painful stimuli [2], and
localizes painful stimuli . The motor examination is symmetric, and the
patient appears to be sensate in all extremities. His reflexes are 2+
bilaterally throughout the upper and lower extremities with down going
toes. Laboratory studies reveal:
a leukocytosis of 24,000/mm3 with a left shift, and are otherwise
unremarkable. A CT scan is completed which shows no mass, shift,
bleed, or edema.
1 . What is the most likely diagnosis?
2 . What is the next diagnostic study of choice?
3 . What is the most appropriate treatment of this condition?
4 . How could we prevent this condition ?
Problem 8
Poisoning
Objectives
1. Recognize the clinical manifestations of Paracetamol intoxication.
2. Know the treatment for acute Paracetamol intoxication.
Case history
An 18-year-old woman is brought by her mother to the emergency
department (ED)about 30 minutes after she took “a bunch” of
Paracetamol . The patient states she was upset with her brother who
grounded her after she came home late from a party with her female
friends ; she swallowed half a bottle of extra-strength Paracetamol in
order to “make them feel sorry.” She is tearful, says she was “stupid,” and
denies any true desire to hurt herself or anyone else. She has no other
complaints and denies any past attempts to hurt herself. On examination,
her blood pressure is 105/60 mm Hg, heart rate is 100 beats per minute,
and respiratory rate is 24 breaths per minute (crying).
Her pupils are equal and reactive bilaterally. Her sclera is clear, and her
mucous membranes are moist. The lungs are clear and the heart sounds
are regular. The abdominal examination is benign with normal bowel
sounds. She is awake and alert without any focal neurologic deficits.
1 . What is the most appropriate next step?
2 . What are the potential complications of this ingestion?
3 . What is the mechanism of acetaminophen toxicity?
4 . She was discharged home after 24 hour stay but unfortunately she
returned 3 days later with jaundice , abdominal discomfort and confusion
, what happen to her and how you are going to manage her ?
5 . If the patient develops life threatening liver failure how could you
manage her ?
Problem 9
Sickle cell crises
Objectives
1. Recognize the clinical signs and symptoms of sickle cell crisis and its
associated complications.
2. Understand the diagnosis and treatment of acute chest syndrome.
3. Understand the treatment of pain crises in patients with sickle cell
disease.
A 30-year-old woman with a history of sickle cell disease presents to the
emergency department (ED) complaining of chest pain for 2 days. She
states that the pain is right sided, worse with inspiration, and is more
severe than her usual “crisis pain.” She has subjective fevers, mild
shortness of breath, and a productive cough. She denies vomiting,
hemoptysis, or lower extremity swelling. Her last pain crisis was 3
months ago. She usually takes acetaminophen and hydrocodone for pain
control during her crises, however, neither have been able to provide
sufficient relief during this episode. On physical examination, her
temperature is 38.3 C (101 F), blood pressure is 130 /70 mm Hg, heart
rate is 98 beats per minute, respiratory rate is 22 breaths per minute, and
oxygen saturation is 94% on room air. Lung examination reveals crackles
in the right lower lung field. She does not have any jugular venous
distension, calf tenderness, or lower extremity edema.
1 . What is the most concerning issue regarding this patient?
2 . What is the initial management?
3 . Enumerate five other type of crisis in this condition ?
4 . Is there any long term treatment for this condition ?
5 . How would you approach the question of genetic counseling for
someone with sickle disease ?
6 . What is Hb SC disease ?
7 . Mention the characteristic bone changes in this patient's X-ray
Problem 10
Cold Fingers in connective tissue diseases
Case history
A secretary aged 50 complained that her fingertips had become so painful
that she could no longer type . Since childhood her fingers had blanched
on exposure to cold and she described the the typical color changes of
Raynaud's phenome non. During the year prior to presentation her fingers
had remained persistently cold & purplish in colour and had become
swollen . Painful ulcers had developed on tips of several fingers . She had
history of heart burn attributed to occasional difficulty in swallowing .
Heart burn relived by using excess pillows .
O/E appeared well . The skin of her hands cyanosed and swollen. There
was pulp atrophy and terminal pitting ulceration of several fingers . The
skin of the dorsum of the hands is also thickened . Her facial appearance
was normal as was the remainder of physical examination . BP 130/75
mmHg .
Investigations :
ESR 30 mm/1st hr CBP MCV normal Renal LFT, Electrolytes normal
Total Protein 78g/L , Albumin 40 g/L , Ca & P normal
Urine analysis normal
1 . What is the differential diagnosis ? Give 3 D Dx.
2 . What further investigations do you consider to reach a diagnosis ?
3 . How would you manage this patient ?
4 . What form of liver disease this patient might develop in future ?
5 . During the course of follow up of the patient , he present with nausea ,
vomiting , abdominal discomfort and marked abdominal distension .
What is this complication ?
Problem 11
Headache due to CNS disorders
Objectives
1. Learn to differentiate emergent, urgent, and less urgent causes of
headache.
2. Understand the treatment of various types of headache.
Case history
A 50-year-old woman presents with a severe headache of abrupt onset of
10 hours duration. The pain is diffuse, throbbing, and worsened when she
went outside into the sunlight. She denies any recent fever, neck pain,
numbness, weakness, vomiting, and any change in vision. She was
concerned because she has never had a headache as strong as this before.
Her past medical and family histories are unremarkable. She does not
take any medications, does not smoke,
On examination, her temperature is 36.9°C (98.4°F), blood pressure
140/70 mm Hg, heart rate 88 beats per minute, and respiratory rate of 16
breaths per minute. She is not in any acute distress but appears to be
mildly uncomfortable. Her pupils are equal and reactive bilaterally. There
is no evidence of papilledema on fundoscopic examination. Movement of
her neck causes her some increased discomfort. Her neurological
examination is normal, including cranial nerves, strength, light touch
sensation, deep-tendon reflexes, and finger-to-nose.
The computed tomography (CT) scan of her head is normal.
1 . What is the most likely diagnosis?
2 . What is the next diagnostic step?
3 . What are the three possible causes of this condition ?
4 . This lady during follow up you discover to have bilaterally enlarged
kidneys and you decided to refer her to a neurosurgeon Why ?
Problem 12
Upper vs Lower UTI infection
Objectives
1. Recognize the clinical signs and symptoms of UT Is.
2. Understand the diagnosis and treatment of UTIs.
3. Understand the spectrum of UTIs and their variable treatment.
Case history
A 24-year-old woman presents to the emergency department (ED) with
complaints of flank pain and fever for the last 1 to 2 days. She describes
feeling pain with urination over the previous week. She is currently
feeling febrile and nauseated, but has not vomited. The pain in her right
flank is a dull, constant, non-radiating ache that she rates as 5/10 for pain.
She took 600 mg of ibuprofen last night to help her sleep, but this
morning the pain persisted so she came into the ED for evaluation. She
reports that she is sexually active and her last menstrual period was 1
week ago. She denies any vaginal discharge or abdominal pain. Her vital
signs include a temperature of 38.3°C (101°F), heart rate of 112 beats per
minute, respiratory rate of 15 breaths per minute, and blood pressure of
119/68 mm Hg. Her examination is significant for tenderness to palpation
on her right costovertebral angle .
1 . What is the most likely diagnosis?
2 . What is the best treatment?
3 . mention the important risk factors for development of this condition
4 . Mention the appropriate investigation of this patient in order to
prevent future recurrence of this condition
Problem 13
Anemia
Background
As in any other clinical scenario, the investigation of this patient hinges
on an accurate history and examination. In terms of causes of anemia, the
most common by far is iron deficiency and the likelihood of this as an
explanation should be fairly clear by paying close attention to diet
history, enquiring for gastrointestinal symptoms and obtaining
gynecological history in females. A lot of information can be gleaned
from the full blood count result, the key parameter being red blood cell
(RBC) size. In terms of etiology, the classification of anemia (Figure 1.1)
is best approached based around the MCV. Clearly in uncomplicated iron
deficiency, patients will have a microcytic hypochromic blood
film, and it is debatable whether such patients need referral to a specialist
hematology clinic. The common causes of iron deficiency are related to
occult gastrointestinal blood loss or menorrhagia and the appropriate
management of such cases lies with the appropriate specialty. However,
this case presents one of the commonest diagnostic dilemmas – the
patient with a normocytic anemia in whom hematinic levels are normal
and there is no morphological evidence of bone marrow failure or
hemolysis. It is a common problem, the presence of which increases with
age. Often patients are asymptomatic, the condition being discovered by
‘routine’ laboratory tests. Although all causes of microcytic and
macrocytic anemia can on occasion present with a normocytic anemia,
often in this situation one is dealing with a decreased production of
normal-sized RBCs, as occurs in the anemia of chronic disease (ACD)
but also in aplastic anemia. A similar appearance can occur in an
uncompensated increase in plasma volume, for example in pregnancy or
fluid overload. Anemia of chronic disease is thought to be the most
common form of normocytic anemia and probably the second most
common form of anemia worldwide after iron-deficiency anemia. It is
often a disease of exclusion and the pathogenesis is usually multifactorial
and thought to be related to hypo activity of the bone marrow with
relative inadequate production of erythropoietin (EPO) or a poor response
to EPO.
There is probably also reduced RBC survival. The clinical history and
examination will often identify the underlying chronic disorder, which
includes a wide range of inflammatory
conditions including arthritis, infections and malignant disease. Iron
studies usually reveal a decreased serum iron level but with a decreased
transferrin level (rather than a raised level as seen in iron deficiency) and
a normal or elevated serum ferritin.
GiFAbs, growth inhibitory factor antibodies; Hb, hemoglobin; thal,
thalassemia; WCC, white cell count.
Treatment
Treatment of this condition is troublesome. The adage usually is that
attention be given to the underlying disorder. In the very specific area of
ACD associated with renal disease, which is more appropriately classified
as a separate cause of a normocytic anemia, there is a relative
underproduction of EPO and EPO therapy can transform the quality of
life of these individuals. However, there are several reports in the
literature of the effectiveness of EPO in ACD not associated with renal
disease, with some reports suggesting that routine iron supplementation
may enhance the response.
Conclusion
The possible causes of anemia in a middle-aged lady are many but iron
deficiency remains the most important one to exclude and appropriately
investigate (particularly for occult gastrointestinal neoplasm) if this is
suspected, despite the presence of any coincidental inflammatory
disorder. Anemia of chronic disease is often a diagnosis of exclusion,
but it is a very frequent clinical problem with no clear-cut way forward in
terms of management other than addressing the underlying clinical
problem. However, the likelihood of pharmacological intervention in the
future (other than with EPO therapy) may be greater with the developing
understanding of the mechanisms involved.
Objectives
1.To know the types of anemia
2. To know the clinical approach to managing this patient
Case history
A 55-year-old woman is referred to the hematology outpatient department
by her General Practitioner. She consulted him because of symptoms of
profound fatigue and dyspnea on exertion and a routine full blood count had
revealed her to be anemic. Hemoglobin was 8.3 g/dl, mean cell volume
(MCV) 82 fl, white cell count 5.6 (neutrophils 3.5, lymphocytes 2.1) × 109/l
and platelets 287 × 109/l. Her doctor could not establish a clear-cut cause for
this lady’s anemia and has sought further advice.
1.What important points you need to ask in the history
2. How can physical examination help you to know the cause
3.How would you investigate
Problem 14
Bleeding tendency
Background
A thorough, detailed history of the bleeding is vital in the assessment of
patients with potential hemostatic defects, and it will impact on the extent
of investigation. The duration of bleeding/bruising is important when
considering whether the patient is likely to have a congenital or acquired
hemostatic disorder. Bleeding since childhood, such as menorrhagia since
menarche, would usually suggest a congenital disorder; however,
mild congenital disorders of coagulation may present in adult life. A
family history of abnormal bleeding clearly would also support the
presence of a congenital defect. It is useful to establish the details of
bruising, in particular whether it is spontaneous, occurs after minimal
trauma or after surgery, and its extent. Tonsillectomy and dental
extraction in particular pose significant hemostatic challenges
and if these have not been associated with significant bleeding, a
congenital disorder of hemostasis is much less likely.
Bleeding or bruising on multiple occasions and from different sites merits
more extensive investigation as this suggests a systemic haemostatic
defect. Repeatedly bleeding from one site, such as epistaxis from the
same nostril, suggests a structural abnormality. Mucocutaneous bleeding
(epistaxis, bruising, menorrhagia, gastrointestinal bleeding)
as in this case suggests a disorder of platelet function.
Coagulation factor deficiencies are more likely to be associated with
haemarthrosis, muscle hematoma and post-operative bleeding.
Although menorrhagia is a common gynecological symptom, a specific
cause is identified in less than 50% of affected women. Studies have
shown that bleeding disorders are found in a substantial proportion of
women with menorrhagia and a normal pelvis examination. Inherited
bleeding disorders have been diagnosed in 17% of such patients,
with von Willebrand’s disease being the most common abnormality.
Systemic illness, in particular uremia and liver disease, may cause
abnormalities of hemostasis (Table 2.1).
Table 2.1 Causes of platelet disorders
A detailed drug history is clearly relevant; aspirin and non-steroidal antiinflammatory
agents are common culprits causing platelet function defects, but many
drugs are
implicated (Table 2.2).
Table 2.2 Drugs and other agents known to interfere with platelet
function
Clinical examination should focus on any bleeding, bruising or evidence
of underlying systemic illness, particularly liver disease or a primary
hematological disorder.
This lady has significant menorrhagia and marked bruising. She is not
taking any medication and has no family history of abnormal bleeding,
although her mother died in her thirties.
Investigation strategy
The patient’s history is suggestive of a platelet disorder or von
Willebrand’s disease. VonWillebrand factor (VWF) is a plasma protein
that mediates platelet adhesion to the subendothelium, mediates platelet
aggregation and serves as a transport protein for factor VIII.
When vessel wall damage occurs, platelets undergo many responses
including adhesion, release reactions, aggregation, exposure of a
procoagulant surface, formation of micro particles and clot retraction.
These changes lead to rapid formation of a hemostatic plug at the site of
damage, which prevents blood loss. Defects in any of these functions
and/or platelet number usually cause impaired hemostasis and increased
risk of bleeding.
Baseline investigations should include a full blood count to look for
thrombocytopenia and a blood film. The blood film is important to look
for evidence of a primary hematological disorder such as myelodysplasia,
to assess platelet morphology and to show any abnormalities suggestive
of hepatic or renal dysfunction.
A basic coagulation screen should be performed; this should include
measurements of the prothrombin time, activated partial thromboplastin
time and fibrinogen.
If her history was not particularly suggestive of a bleeding tendency, then
the above investigations are probably sufficient.
However, her history is convincing despite these initial tests being
normal, hence she requires further investigation. The next step would be
to screen her for von Willebrand’s disease and assess her platelet function
using a platelet function analyser (PFA-100® system).
Depending on the results she may require further assessment of her
platelet function by performing platelet aggregometry, platelet
glycoprotein analysis and platelet nucleotide analysis (Figure 2.1).
Figure 2.1 Investigation of easy bruising. DIC, disseminated
intravascular coagulation; PFA, platelet function analyser.
Conclusion
Investigation of the patient with easy bruising or abnormal bleeding
requires a structured approach working through diagnostic tests in a
logical, stepwise order.
Emphasis on the importance of taking a detailed and thorough bleeding
history cannot
be stressed more. It is important to continue to investigate the patient
despite normal initial investigations if the history is highly suggestive of a
hemostatic defect. The PFA is superior to the bleeding time and is a
promising screening tool for investigation of platelet disorders, although
it is not 100% sensitive for all platelet disorders. Additional
clinical applications for the PFA are under development.
Objectives
1.To know the mechanism responsible for adequate hemostasis
2. to know the approach for patient with bleeding tendency
Case History
A 28-year-old woman is referred by her General Practitioner as she is
complaining of easy bruising and menorrhagia.
What further information ( regarding history and physical examination)
would you like?
2.What are the important factors for adequate hemostasis?
3.How can you differentiate clinically between bleeding due to defect in
platelets or coagulator factor defect?
3. How do you investigate this patient ?
Problem 15
Pyrexia of unknown origin
Objectives
1.To know the definition of P.U.O
2. To know the possible causes
3. Know the approach to patient with P.U.O
Case history
A 32-year-old man is readmitted to a tertiary care hospital because of
persistent fever. The story is complicated. He was admitted to the same
hospital 3 months ago with an evening rise of temperature of 2 months’
duration. His temperature had risen to a maximum of 38.9°C and was
associated with night sweats. He had lost 5 kg of body weight during the
previous 2 months, but had no other symptoms except mild anorexia. He
said that there were no cats and dogs in the family. There were no
positive findings on physical examination. Extensive investigations were
carried out .The patient was treated empirically with a four-drug regimen
of antituberculosis chemotherapy. Not only did the fever persist, but he
also developed clinical jaundice after 6 weeks of anti-TB chemotherapy.
The jaundice, however, disappeared after stopping the drugs. A detailed
history taken on this admission does not reveal any additional
information.
1.What would your differential diagnosis include before examining
the patient?
2.Physical examination reveals that the patient is pale but not icteric. His
axillary temperature is 38.1°C and he is sweating profusely; he appears
chronically ill and thin. The liver edge is palpable 1 cm below the right
costal border and is mildly tender; the spleen is also palpable 1.5 cm
below the left costal margin. There are no peripheral lymph nodes. A
grade 2/6 ejection systolic murmur is heard best at the lower left sternal
border. The remainder of the examination is normal.
2.Has examination narrowed down your differential diagnosis?
Further investigations. Chest radiography and transoesophageal
echocardiography are normal. Abdominal ultrasonography reveals mild
splenomegaly with a small rounded hypoechoic lesion (19 mm) within
the spleen. The liver is also enlarged with multiple rounded hypoechoic
lesions of varying sizes involving both lobes. There are also multiple
paraaortic,pre-aortic, precaval, retrocaval, mesenteric and peri-pancreatic
lymph nodes .A rounded hypoechoic space-occupying lesion near the
uncinate process of the head of the pancreas raises the possibility of a
primary malignant tumour in the pancreas. A small amount of fluid is
also seen in the pelvis. A contrast-enhanced computed tomogram (CT) of
the abdomen and pelvis confirms the ultrasound findings, with peripheral
enhancement of the lesions in the liver and spleen. A CT of the chest is
normal.
3.Has the diagnosis been clinched?
An ultrasound-guided FNAC from a retroperitoneal lymph node reveals
predominantly small lymphocytes admixed with binucleate Reed–
Sternberg cells, which have enlarged vesicular nuclei and prominent
nucleoli . Therefore, the cytomorphological features in this patient are
those of Hodgkin’s lymphoma and imaging investigations indicate stage
IV B disease.
4.How will you treat this patient?
This patient requires combination chemotherapy with 6–8 cycles of either
ChIVPP (chlorambucil, vinblastine, procarbazine and prednisolone) or
ABVD (Adriamycin, bleomycin, vinblastine and daunorubicin). The
latter regimen is more potent, but is also more expensive. The patient
should be informed about the side-effects of this chemotherapy. The stage
of the disease, along with the presence or absence of systemic symptoms,
influences the response to treatment. In the UK, a young man such as this,
with extensive disease at presentation and treated with an ABVD
regimen, would have a >80% 5-year survival. The treatment response
should be evaluated by clinical criteria, as well as by repeat CT
examination. Positron emission tomography (PET), if available, is the
most sensitive means of documenting
Problem 16
Malaria
Objectives
1.To give a differential diagnosis of fever of short duration.
2.To know what are the physical signs of help.
3.To know how to investigate.
4. To know how to treat
Case history
A 30-year-old farmer is brought to an Emergency Department in Delhi on
a hot July afternoon with a high-grade fever and altered conscious level.
He was found in a disoriented state by co-workers in the field. His wife
reports that he is normally very fit and well, but that he has had a very
high temperature for the past 8 days and was febrile when he went out to
work this morning. She gives no history of severe headache, loss of
consciousness or seizures.
1.What would your differential diagnosis include before examining
the patient?
Examination
The patient’s rectal temperature is recorded to be 40°C and his skin is hot
and dry. His Glasgow Coma Scale (GCS) is 7, but there is no meningism
and no focal neurological signs. Pulse rate is 120/min and blood pressure
is 94/52 mmHg. The spleen is palpable 2 cm below the costal margin.
The rest of the examination is normal.
The patient’s clothes are removed, his entire body is sprayed with water
in front of a fan and ice packs are applied. After a venous blood sample is
taken, intravenous fluids are started. With this treatment, the patient’s
temperature normalises in 45 min, but there is only mild improvement in
his conscious level (GCS 10).
2.Has examination narrowed down your differential diagnosis?
Investigations
Blood cultures and a Widal test are negative. Magnetic resonance
imagining (MRI) of the brain and cerebrospinal fluid (CSF) examination
are normal. CSF testing
for Japanese B encephalitis virus is negative. Thick and thin peripheral
blood smears show high levels (>5% infected red blood cells) of ring
forms of Plasmodium falciparum.
3.Does this narrow down your differential
diagnosis?
4.How will you treat this patient?
Problem 17
Chronic Obstructive Pulmonary
Disease (COPD)
Background
Chronic obstructive pulmonary disease is estimated to affect three million
people in the UK but is often not diagnosed until the patient is >50 years
old. The principal cause is smoking. COPD is characterized by airflow
obstruction that is not fully reversible and is usually progressive in the
long term. Intermittent exacerbations cause rapid worsening of symptoms
beyond normal day-to-day variation. COPD is defined as airflow
obstruction with a reduction in the post-bronchodilator FEV1/FVC ratio
to less than 0.7. Surprisingly, disability in COPD is often poorly reflected
by the FEV1. If the FEV1 is ≥80% a diagnosis of COPD should only be
made in the presence of respiratory symptoms (e.g. breathlessness, cough,
reduced exercise tolerance, exacerbations) and formal lung function
testing. As in this patient, airflow obstruction is irreversible following
bronchodilator and steroid therapy (i.e. <15% increase in FEV1),
excluding a diagnosis of asthma. The severity of COPD is classified by
(i) post-bronchodilator FEV1 as a percentage of predicted (e.g. National
Institute for Health and Clinical Excellence [NICE] guidelines:1 mild
≥80%; moderate 50%–79%; severe 30%–49%; and very severe <30%
predicted) and (ii) by clinical picture (e.g. exercise capacity,
breathlessness, PaO2 and PaCO2 in arterial blood gases, health status,
body mass index [BMI], frequency of exacerbations, carbon monoxide
transfer and severity of symptoms).
There are two pathophysiological types of COPD:
Emphysema occurs following alveolar septa and capillary destruction due
to inadequate antiprotease defences against inhaled toxins. Tobacco
smoke causes centrilobular emphysema with predominantly upper lobe
involvement, whereas a1-antitrypsin deficiency causes panacinar
emphysema affecting the lower lobes. Lung function tests demonstrate
impaired diffusion (DLCO, KCO) and increased lung volumes (total lung
capacity, FRC, RV). Resting blood gases are often normal because
alveolar septa and capillaries are destroyed in proportion to each other.
Desaturation during exercise and increased minute ventilation are due to
reduced diffusion capacity. CXR reveals hyperinflation (e.g. flat
diaphragms), narrow mediastinum, bullae, hyperlucency and reduced
peripheral vascular markings. Emphysematous patients tend to be
relatively well oxygenated but breathless and cachectic with barrel chests
due to hyperinflation. They are classically described as ‘pink puffers’.
Chronic bronchitis causes airways obstruction due to chronic mucosal
inflammation (± airways damage), bronchospasm and mucous gland
hypertrophy with hypersecretion, with the lung parenchyma less affected.
Diffusion capacity and lung volumes are usually normal. The CXR shows
less hyperinflation but increased vascular markings compared with CXR
findings in emphysema. Ventilation-perfusion (V/Q) mismatch occurs
due to poor alveolar ventilation as a result of early expiratory airways
closure or alveolar collapse following airway obstruction. Breathlessness
is unusual due to the development of brainstem insensitivity to PaCO2.
Consequently these patients are often hypoxaemic, hypercapnic and
oedematous and are described as ‘blue bloaters’. However, the concept of
emphysematous ‘pink puffers’ and bronchitic ‘blue bloaters’ is outdated
as most patients have coexistent elements of both disease processes and a
mixed clinical picture (i.e. ‘blue puffers’).
Non-pharmacological measures for COPD therapy
*** Smoking cessation is the single most important intervention to
prevent COPD and stop its progression. Even a three-minute period of
counselling urging a smoker to ‘quit’ in a clear, strong and personalized
manner can be effective and this should be done for every smoker at
every visit. Further success is reported with support schemes (e.g. Action
on Smoking and Health [ASH], National Health Service [NHS]) and anti-
smoking pharmacotherapy (e.g. nicotine replacement therapy, bupropion,
varenicline).
*** Vaccination against influenza and pneumoccal infection is essential.
In COPD patients, influenza vaccination reduces serious illness and death
due to influenza by about 50%.
*** Nutrition. BMI should be monitored and maintained in the range >20
kg/m2 and <25 kg/m2. When BMI is <20 kg/m2, weight gain improves
respiratory muscle strength and prognosis. Similarly, when BMI is >25
kg/m2, reducing weight improves functional status and reduces dyspnoea.
*** Pulmonary rehabilitation reduces symptoms, strengthens respiratory
muscles, increases exercise tolerance, reduces hospitalizations, enhances
quality of life and increases physical and emotional participation in
everyday activities. These benefits are achieved by education (e.g.
breathing techniques), addressing altered mood, reducing social isolation
and by preventing weight loss, muscle wasting and exercise
deconditioning.
Bronchodilators: mechanisms of action
Inhaled bronchodilators are central to the symptomatic management of
COPD. Although spirometric improvements are small, studies show
reductions in FRC (i.e. hyperinflation) that correlate well with improved
functional status, dyspnoea index and quality of life measures. Because
B2 agonists and anticholinergic agents have different mechanisms of
action, acting on the sympathetic and parasympathetic systems,
respectively, they act synergistically when used in combination. B2
agonists have a rapid onset of action (<5 minutes) and are initially
prescribed as short-acting agents (e.g. salbutamol). As COPD severity
progresses, LABA (e.g. salmeterol, formoterol) are used and further
improve health status. Anticholinergic agents (i.e. muscarinic receptor
antagonists) have a slower onset of action (maximum effect 30–40
minutes) but are equally effective (or better) than B2 agonists. Both shortacting (e.g. ipratropium bromide) and longacting (e.g. tiotropium
bromide) anticholinergic agents improve health status and reduce
hyperinflation. However, once-daily tiotropium bromide is superior to
ipratropium bromide as it acts specifically on the M3 muscarinic receptor
to produce bronchodilation, whereas ipratropium bromide acts on three
receptors, where effects at M1 and M3 produce bronchodilation but
action at M2 causes bronchoconstriction. Consequently ipratropium could
potentially neutralize the benefit of tiotropium and these two drugs should
never be used in combination. Compared to ipratropium, tiotropium
reduces COPD exacerbations by about 20% and hospital admissions by
approximately 40%.
Effectiveness of inhaled corticosteroids in COPD
In the ISOLDE Trial, ICS did not reduce the rate of decline in FEV1 in
COPD patients when compared with placebo. However, ICS may reduce
exacerbations and improve lung function, both alone and in combination
with LABA, although this has not been found in all trials. The TORCH
study, a double-blind, prospective analysis of COPD survival in 6100
patients, demonstrated a reduction in exacerbations and hospital
admissions and improved quality of life in patients treated with
combination ICS/LABA therapy compared to ICS or LABA alone or
placebo. In the ISOLDE Trial, the reduction in exacerbations was greatest
in patients with an FEV1 <1.25 l. Subsequent post hoc analysis of the
ISOLDE data and a number of retrospective database analyses of ICS and
LABA combination therapy suggest a small survival benefit. In
particular, a retrospective cohort study from the UK General Practice
Research Database demonstrated that regular LABA (salmeterol) and/or
ICS (fluticasone propionate) significantly improved three-year survival
compared to controls (79% vs 63%; P <0.05).
Long-term ICS in mild/moderate COPD (i.e. FEV1 >50% predicted) is
only recommended if a previous trial of inhaled or oral steroids has
shown a beneficial spirometric response. Long-term treatment with oral
corticosteroids is not recommended in COPD; it benefits less than 25% of
patients and steroid-induced myopathy and other systemic side effects
may contribute to muscle weakness and early respiratory failure.
Other helpful pharmacological therapies in COPD
*** Methylxanthines (e.g. theophylline, aminophylline) may be
prescribed as slow release oral preparations in stable COPD after trials of
short- and long-acting bronchodilators. Plasma levels must be monitored
at higher doses due to the narrow therapeutic index. Theophylline can
improve exercise tolerance, respiratory drive, diaphragmatic strength,
mucociliary clearance and blood gases but the effect on spirometry is
negligible.
*** Mucolytics therapy may be beneficial in patients with chronic cough
productive of excessive sputum. If a short trial is not of symptomatic
benefit, treatment should be stopped.
*** Conventional antidepressant pharmacotherapy should be considered
in severely depressed patients.
*** Benzodiazepines and narcotics are respiratory depressants and may
worsen hypercapnia. They are not recommended except as palliative
measures in end-stage COPD when they may help relieve severe
dyspnoea.
*** Diuretics (but not angiotensin converting enzyme inhibitors or
calcium channel blockers) should be considered in patients with
peripheral oedema due to pulmonary hypertension (i.e. cor pulmonale).
Therapies that are not recommended in stable COPD include antitussive
agents, as cough has a significant protective effect; a1-antitrypsin
replacement
antioxidants;
therapy
in
respiratory
patients
with
stimulants;
a1-antitrypsin
regular
deficiency;
antibiotics
and
immunoregulators.
The criteria for long-term oxygen therapy
Long-term oxygen therapy (LTOT) is only indicated in patients with a
PaO2 <7.3 kPa when stable or a PaO2 >7.3 kPa and <8.0 kPa when stable
with at least one of the following: secondary polycythaemia, nocturnal
hypoxaemia (SaO2 <90% for 30% of sleep time), peripheral oedema or
pulmonary hypertension. Blood gases should be assessed on two
occasions at least three weeks apart in stable patients receiving optimum
therapy. However, ambulatory oxygen should be considered in patients
with exercise desaturation (as in this case) and those with reduced
breathlessness or improved exercise capacity with oxygen.
Conclusion
This patient has severe COPD (i.e. FEV1 35% predicted), predominantly
emphysema as indicated by the alveolar air trapping (i.e. raised RV and
FRC), CXR hyperinflation, and reduced KCO due to loss of alveolar
surface for gas transfer with associated exercise desaturation.
His management can clearly be improved and smoking cessation should
be encouraged. He should be referred to a smoking cessation service for
pharmacological and psychological support during attempts to stop
smoking as ‘quit rates’ are improved. Pharmacological management
should include the addition of tiotropium bromide and LABA inhalers.
The LABA should be combined with an ICS as he has been treated with
three courses of inhaled steroid in the last year. He may also benefit from
pulmonary rehabilitation and encouragement to improve his nutritional
status. Finally, portable oxygen may improve his exercise tolerance but
ideally this should only be considered if the patient has stopped smoking
due to the associated fire risks.
Objectives
1. To know the pathology of C.O.P.D
2. To Know the symptoms and signs of the disease
3. To know the result of investigations
4. To know how to treat
Case History
A 65-year-old man with history of COPD and increasing breathlessness
was referred for review of his management. He has limited exercise. He
was treated with full-dose inhaled ipratropium bromide and was using his
salbutamol inhaler >10 times a day. Three COPD exacerbations in the
last year had been treated with steroids and antibiotics. He was a lifelong
smoker of 20 cigarettes a day.
On examination he was cachectic, barrel-chested and comfortable at rest
but breathless on minimal exertion with a resting saturation (SaO2) of
91%. He had minimal ankle oedema but the jugular venous pressure was
fractionally elevated at 2 cm. Chest examination revealed poor air entry
to both lungs and widespread soft expiratory wheeze. Spirometry
recorded a forced expiratory volume in 1 second (FEV1) of 0.9 l (35%
predicted), forced vital capacity (FVC) of 2.6 l (74% predicted) and peak
expiratory flow rate of 120 l/min. Recent oral steroid and bronchodilator
trials had increased his FEV1 by 4% and 3%, respectively. A chest
radiograph (CXR) revealed hyperinflation with small apical bullae and a
cardiothoracic ratio at the upper limit of normal. Formal lung function
tests demonstrated a residual volume (RV) of 198% predicted and a
functional residual capacity (FRC) of 5.2 l or (184% predicted). Carbon
monoxide gas transfer (TLCO) corrected for alveolar volume (KCO) was
42% of predicted. Desaturation occurred during exercise testing, and
exercise tolerance and dysnoea improved with ambulatory oxygen.
1. How is COPD defined and how severe is it in this patient?
2. Explain the physical signs of this patient
3. Explain the results of pulmonary function test
4. What do you expect the result of arterial blood gases
5. What non-pharmacological measures may help this patient?
6. Which bronchodilators are most effective and why?
7. Are inhaled corticosteroids effective in COPD?
8. What other pharmacological therapies may be helpful in COPD?
Problem 18
Massive Pulmonary Embolism
Back ground
Massive PE is characterized by persistent hypotension, usually defined as
a systolic blood pressure <90 mmHg. A fall in systolic blood pressure of
at least 40 mmHg for greater than 15 minutes is also significant. It is
important to exclude an acute myocardial infarction and tension
pneumothorax. Massive PE can lead to acute right heart failure; death is
usually within 1–2 hours of the embolism.
Investigations
A CTPA has been shown to be an accurate diagnostic tool for pulmonary
thromboembolic disease, with higher interobserver con- cordance than
ventilation-perfusion (V/Q) scanning. Evidence shows that sensitivity
and specificity with CTPA are higher than V/Q scanning, and that a
negative CTPA is as good as a negative or low-probability V/Q scan to
rule out PE. There is a debate over whether some smaller subsegmental
pulmonary emboli that are found on a CTPA actually require treatment.
Magnetic resonance imaging has not yet been found to be any more
useful than traditional imaging.
Bedside echocardiography can be useful to assist in the diagnosis of PE
when the patient is too unwell to undergo a CTPA. A real-time
echocardiogram may show right ventricular dysfunction, tricuspid valve
regurgitation and possibly thrombus in the right ventricle.
The d-dimer is a non-specific indicator of the presence of
thromboembolic disease. It measures the by-product of intravascular
fibrinolysis. A positive d-dimer result informs you that the process of clot
formation and degradation may be taking place, but does not tell you
where or in what circumstances. As a test for PE or deep vein thrombosis
(DVT) the d-dimer is only really useful when negative. A positive ddimer test has a high sensitivity for thromboembolic disease; however, its
positive predictive value is low, with many positive results in the absence
of proven PE or DVT.
The Wells’ score is a clinical probability score useful for excluding PE
and DVT without further investigations (Table 1). A PE can usually be
excluded if the patient has a low clinical probability score and a negative
d-dimer test. However, in cases of moderate to high clinical probability, a
negative d-dimer result cannot be relied upon to exclude PE. Clinical
probability scoring is now commonplace as part of the work-up for PE.
The Wells’ score is not intended to be used in post-operative and longterm inpatients.
Table 1 Wells’ criteria for assessment of
pre-test
probability
for
pulmonary
embolism
Criteria
Points
Suspected DVT
3.0
An alternative diagnosis is less likely than
PE
3.0
Heart rate >100 beats per minute
1.5
Immobilization or surgery in the previous
four weeks
1.5
Previous DVT or PE
1.5
Haemoptysis
1.0
Malignancy (on treatment, treated in the
last 6 months or palliative)
Score
Mean
range
probability
<2
1.0
Risk
of pulmonary
PE %
embolism
3.6
Low
20.5
Moderate
points
2–6
points
of
>6
66.7
High
points
Cardiac troponin as evidence of myocardial damage is a strong
prognostic marker in acute PE. An elevated serum troponin correlates
with right ventricular dysfunction on a ‘same-time’ echocardiogram;
higher troponin rises are also predictive of increased inhospital mortality,
post-PE complications and recurrent PE. In this case the troponin rise
suggested that this patient was at greater risk of cardiovascular instability
in the acute setting and that he might be at risk of future thromboembolic
disease. Brain natriuretic peptide (BNP) is a useful but less commonly
utilized marker of cardiomyocyte stretch and shear stress in acute PE.
Duplex venous ultrasound of the lower limb may assist in the diagnosis
of PE. Residual lower limb DVT can be seen in 30%–50% of patients
with a PE.
Treatment of pulmonary embolism
The majority of patients with a proven PE are anticoagulated, usually
with low molecular weight heparin whilst being loaded and stabilized on
warfarin. In the presence of a known temporary trigger, such as a long
bone fracture, anticoagulation for PE can be discontinued after 4–6
weeks. For an idiopathic PE in the absence of any known ongoing risk
factors anticoagulation is usually continued for between three and six
months.
Thrombolysis for massive PE is recommended when there is evidence
of arterial hypotension, cardiogenic shock, hypoxia with a metabolic
acidosis, a reduced conscious level or oliguria. Early mortality in patients
with massive PE is at least 15%, and in these patients the bleeding risks
associated with thrombolysis are acceptable. The Cochrane Collaboration
found that thrombolysis improved haemodynamic outcomes and post-PE
imaging appearances, but had no appreciable benefit in death rate or
recurrence; there was no significant difference in minor or major
bleeding events. Patients should be managed in a high-dependency
setting as they are at increased risk of death for 24–72 hours.
There are a few patients worldwide each year that undergo emergency
open pulmonary thromboembolectomy for acute massive PE; a further
group have this procedure electively as a treatment for chronic post-PE
pulmonary hypertension. It is a high-risk operative procedure; however,
in a specialist centre with appropriate clinical support it has been shown
to have comparable mortality to thrombolysis in the acute setting.
Prognosis
Current guidance on treatment of PE in the absence of ongoing risk
factors is to anticoagulate for three to six months. Lifelong
anticoagulation should be considered in patients with recurrent PE or
DVT. Most patients make a complete recovery after a PE. However, a
minority
(1%–3%)
develop
chronic
thromboembolic
pulmonary
hypertension. PE should be suspected in patients with continued
dyspnoea, signs of pulmonary hypertension and/or echocardiographic
evidence of right ventricular dysfunction. Massive PE is a risk factor for
the development of chronic thromboembolic pulmonary hypertension. A
positive troponin or BNP at six months has been shown to indicate
continued
right
ventricular
dysfunction.
Patients
with
chronic
thromboembolic pulmonary hypertension should receive lifelong
warfarin
and
be
thromboembolectomy.
referred
for
consideration
of
pulmonary
Conclusion
Massive PE is life threatening and should be a potential differential
diagnosis for all acutely unwell patients. If PE is suspected, imaging to
confirm the diagnosis should be arranged immediately and thrombolysis
should be considered within an hour of presentation
Objectives
1.To know the risk factors for pulmonary embolism
2. To know the symptoms and signs of P.E
3. How to investigate
4.How to treat
Case History
A 46-year-old man presented to hospital after he collapsed with brief loss
of consciousness. He was short of breath at rest and felt as if he couldn’t
take a breath in. He felt dizzy on standing. He did not have chest pain. He
had no significant past medical history. He had never smoked. On
examination he was overweight, sweaty and tachypnoeic. His systolic
blood pressure was 88 mmHg and heart rate was 120 beats/min. He was
afebrile. Jugular venous pulse was at 7 cm. Auscultation of his chest
revealed normal breath sounds with no added sounds. Oxygen saturation
was 94% on 10 l/min of oxygen. Arterial blood gas analysis, on oxygen,
revealed a pH 7.46, and partial pressure of oxygen (PaO2) of 9.5 kPa and
carbon dioxide (PaCO2) of 3.9 kPa. His chest radiograph was
unremarkable.
A
twelve-lead
electrocardiogram
(ECG)
showed
borderline ST elevation in the chest leads and T wave inversion in lead V.
D-dimer and troponin were positive. An urgent computed tomography
pulmonary
angiogram
(CTPA)
was
performed.
1. What do you see on the CTPA?
2. What are the risk factors for this condition
2. How do you explain the results of the d-dimer, troponin and ECG?
3. How would you manage this patient?
4. Is there is a role for surgery?
5. For how long you continue treatment?
6. Write short notes about the prognosis?
7. How do you prevent?
.
Problem 19
SYSTEMIC LUPUS ERYTHEMATOSUS
(SLE)
Back ground
SLE is a disease characterized by the production of a variety of antibodies
against nuclear components that causes inflammation and injury to
multiple organs. It primarily affects women from the late teens onwards
with a peak incidence between the ages of 15 and 45 years. In European
populations the prevalence is 12–64 cases per 100 000, and it is three to
five times more prevalent in African-American and Afro-Caribbean
women.
Recent studies in human populations and animal models have associated
elements of the innate immune system and abnormalities in the immature
B lymphocyte receptor repertoires with disease initiation. A variety of
cytokines, most notably type 1 interferons (IFN-a, IFN-b), play an
important role in pathogenesis.
The American College of Rheumatology (ACR) criteria for the
classification of SLE act as an aide-memoire to the multisystem nature of
SLE and to the investigations that should be undertaken. Patients with
SLE almost always suffer from debilitating fatigue, with the majority
having associated weight loss and fever. Three-quarters of patients with
SLE have mucocutaneous disease in the form of butterfly rash, oral
ulcers, alopecia and Raynaud’s phenomenon; purpura, vasculitis or
urticaria occur less often. The majority experience arthralgia or arthritis
in a pattern that may mimic rheumatoid arthritis (RA). Renal disease and
hematological problems occur in approximately one third of patients;
cardiac, neuropsychiatric and gastrointestinal problems occur in 20%.
Lack of antinuclear antibody (ANA) at a titer of 1:160 or higher makes
SLE very unlikely.
Remaining patients are often positive for anti-Ro antibodies. Falsepositive ANA results increase with age, and up to 5% of healthy
individuals may be ANA positive.
Objective
1.To study the disease from clinical, serological, pathogenesis and
prognosis
2.Summarized the SLE-related environmental and risk factors
3.Discussion the approach to diagnosis
4.Discussion the monitoring and managing the disease
5.Relation of the disease and pregnancy
6.Recent development
7.Further readings
Case history
F.A a young lady, aged 21 years, presented with fatigue, a photosensitive
rash on her arms and face, recurrent mouth ulcers and sore joints. She
smokes 20 cigarettes per day. Her medications are the oral contraceptive
and minocycline for acne. Investigations show ESR 50mm/h, HB
9mg/100cc, RF and Anti-ccp ab are both negative. ANA and Ds antiDNA are both positive Urinalysis shows granular cast and protein urea.
1.What are the pathophysiological changes of SLE?
2.What environmental or inherited risk factors predispose to SLE?
3.What are the most frequent manifestations and organ involvement of
SLE?
4.On what basis could you make a diagnosis of systemic lupus
erythematosus (SLE)?
5.What further investigations should be undertaken?
6.What is drug-induced lupus?
7.What are the poor prognostic criteria of SLE?
8.What monitoring should she undergo?
9.What are the current treatments for SLE , general measures and medical
according to the stage of the disease?
What are the impact of the disease on the pregnancy? and neonate?
11.What are the new concepts?
Problem 20
Rheumatoid arthritis (RA)
Background
Rheumatoid arthritis (RA) is the most common persistent inflammatory
arthritis, occurring throughout the world and in all ethnic groups. The
prevalence is lowest in black Africans and Chinese, and highest in Pima
Indians. In Caucasians, approximately 0.8–1.0% are affected, with a
female to male ratio of 3 : 1.
The clinical course is prolonged, with intermittent exacerbations and
remissions. Patients with RA have an increased mortality when compared
with age-matched controls, primarily due to an increased risk of
cardiovascular disease.This is most marked in those with severe disease,
with a reduction in expected lifespan by 8–15 years. Around 40% of RA
patients are registered as disabled within 3 years of onset, and around
80% are moderately to severely disabled within 20 years. Functional
capacity decreases most rapidly at the beginning of disease and the
functional status of patients within their first year of RA is often
predictive of long-term outcome. Factors that associate with a poorer
prognosis are disability at presentation, female gender, involvement of
MTP joints, radiographic damage at presentation, smoking and a positive
RF or ACPA. It is likely that the prognosis of RA will improve as more
effective treatment regimens are introduced in patients with early disease.
In former years, around 25% of patients required a large joint
replacement but rates are now falling, probably reflecting more
aggressive and effective medical therapy.
Objectives:
1.To discuss the pathophysiology ,genetics, immunological and
environmental factors influencing the disease.
2.Clinical manifestations of the disease articular , extra articular features ,
complications and co-morbidities
3.Early diagnosis of the disease
4.Investigations(imaging ,hematological, serological and others)
5.Patients education about the disease in all aspects
6.Managing rheumatoid arthritis at onset(non-pharmologic general
measures, and drug therapy including symptomatic and disease
modifying therapy including advances in biological therapy and surgical
treatment)
7. Evaluating the response to treatment and monitoring disease activity
Pregnancy and RA
Case history
A 25-year-old female presents with intermittent pain and swelling of the
MCP joints and MTP joints for the last 6 months .An x-ray of MCP
shows decrease joint space and soft tissue swelling . On examination she
has synovitis of the MCP ,MTP and PIPjoints with associated joint
tenderness .Labs reveal ESR = 25 mm/h,
-veRF, +ve ant-CCP antibody. CRP is 162 mg/l
1.What genetic factors influence the development or severity of RA?
2.What environmental factors are known to be involved?
3.How does being RF positive or/and anti-ccpantibodies influence the
likelihood of developing and prognosis of RA?.
4.What investigations should be undertaken at the onset of RA?
5.What investigations help with monitoring the activity of the condition?
6.What types of pharmocolgic and non pharmoclogical therapy in treating
articular, extra-articular and disease cormobidities?
How should follow-up be planned?
8.What type of surgery that can performed in treating articular and soft
tissue sequel of RA?
9.How pregnancy influence the disease and what are safe drugs used in
pregnancy and lactation?
What are recent develops?
Problem 21
Osteoporosis
Background
Osteoporosis arises from loss of BMD with consequent disruption of
bony microarchitecture and increased fracture risk. Osteoporosis is, by
definition, present when the T score is –2.5 or less – i.e. bone density is
2.5 standard deviations below the estimated peak BMD for the
population. Osteopenia is defined as a T score between –1 and –2.5.
While both males and females are at risk of fracture in later life, the
dramatic decrease in oestrogen at menopause in women means that they
are generally at greater risk from an earlier age. BMD is most
conveniently measured by dual-energy X-ray absorptiometry
(DEXA). Screening of the population with DEXA is not generally
recommended but may be justified in women aged over 65 years. Low
BMD should always be interpreted in the light of the overall clinical
picture and estimated fracture risk. All patients with fragility
fractures should be screened for osteoporosis, and treatment should be
considered where indicated. Fifty per cent of women and 20% of men
will suffer a fragility fracture during their lifetime. Osteoporotic fracture
is uncommon below the age of 60 years, and 85% of fractures occur in
subjects over the age of 65. Peak bone density is attained in early adult
life (around age 30 years); there is a steady decline in BMD thereafter,
and this accelerates markedly after the menopause. Individuals with
higher peak BMD are better able to withstand the later decline in BMD.
objectives :
1.To summarizing the pathophysiolgic changes in osteoporosis
2.Discussing the etiological backgrounds
3.Evaluating main clinical features and diagnostic tool
4.Maintaining follow up assessment
5.Management : general measures , modification of life style, supplement
treatment and anti-osteoporotic drug therapy
6.Discussing post-menopausal osteoporosis, male osteoporosis,
glucocorticoid-induced osteoporosis, osteoporosis in pregnancy ,
osteoporosis associated with certain diseases.
case history
F.A is a fit 51-year-old woman whose periods are becoming infrequent.
She is concerned about developing osteoporosis as she approaches the
menopause. Her mother has recently fractured her hip. Jane has recently
had her bone mineral density (BMD) measured, and was told that she has
osteopenia.
1.What are the risk factors for osteoporosis?
2.What advice would you give her on preventing osteoporosis?
What is the role for calcium and vitamin D supplementation?
Recent development
Bisphosphonates for Osteoporosis –
Which Agent and When?
Background
Bisphosphonates (BPs) have found wide usage in patients with metastatic
bone disease, myeloma, Paget’s disease and osteoporosis. They are stable
analogues of inorganic pyrophosphate and bind to hydroxyapatite bone
surfaces with high affinity, decreasing bone turnover by inhibiting
osteoclastic activity. risedronate, pamidronate, ibandronate and
zoledronate. These modify the synthesis of isoprenoid compounds by
inhibiting the enzyme farnesyl pyrophosphate synthase. They thus modify
activity of several guanosine triphosphate (GTP)-binding proteins,
inhibiting activity and promoting apoptosis of osteoclasts
Case History
A.A is a 66-year-old woman who has lost 3 cm in height over the past
three years. She has not had any other fractures. Her only medication is a
diuretic that she takes for hypertension. X-ray reveals a wedge fracture of
the T12 vertebra and generalized osteoporosis. A subsequent dual-energy
X-ray absorptiometry scan shows the T score for her lumbar vertebrae to
be –2.8, and that for her hip to be –2.1.
1.Would you carry out any further investigations?
2.Should she have treatment with a bisphosphonate?
3.If so, which one would you choose and for how long should it be used?
Osteoporosis – Drugs Other Than
Bisphosphonate
Back ground
Bisphosphonate drugs have been the cornerstone of osteoporosis
treatment in recent years. However, a small proportion of patients do not
appear to respond. While changing to another, sometimes more potent,
bisphosphonate may be the answer for some patients, other treatment
options are now available like hormonal replacement therapy denosumab,
parathyroid hormone, strontium ran elate and others.
Case History
S.A is a 60-year-old woman who developed a wedge fracture of her L1
vertebra two years ago. She had low bone mineral density (BMD) (T
score –2.6 for lumbar spine, –1.6 for hip). At a recent follow-up dualenergy X-ray absorptiometry (DEXA) scan, her BMD had deteriorated
further (lumbar spine T score –2.9, hip –2.1). She insists that she has been
taking her bisphosphonate, and continues to suffer back pain.
1.Should she persist with bisphosphonate treatment?
2.What other treatments should be considered?
3.Is combination therapy an option?
Male Osteoporosis
Back ground
Although osteoporosis is predominantly a disease affecting females, it
should be remembered that 20% of fractures, and 30% of hip fractures,
occur in males. One in three men over the age of 60 years will suffer a
fracture. Osteoporosis in men is more likely to present with a fragility
fracture, while many cases in women are diagnosed from screening.
Morbidity and mortality following fracture is poorer in men than in
women. For example, 80% of men do not return to their pre-fracture
physical function after hip fracture and as many as 50% will not return to
an independent existence. Underlying causes are more commonly
discovered in men, with glucocorticoid excess (mostly iatrogenic)
occurring in 20% of cases, heavy alcohol intake in 15%–20% and
hypogonadism in 15%–20%. Other secondary causes such as
hyperthyroidism, hyperparathyroidism, malabsorption, antiepileptic
treatment and multiple myeloma should be considered as for female
osteoporosis .
Case History
A 60-year-old man presents with back pain. X-ray of his spine shows a
crush fracture of L2 and a generalized decrease in bone mineral density
(BMD). A subsequent dual-energy X-ray absorptiometry (DEXA) scan
confirms that he has osteoporosis with T scores of –3.2 for the lumbar
spine and –2.6 for the hip. His serum testosterone is 7.2 nmol/l (normal
range 9–30 nmol/l).
1.What are the causative factors in male osteoporosis?
2.What investigations should routinely be carried out?
What treatment options are proven to work?
Glucocorticoid-Induced Osteoporosis
Background
Overall, 0.5%–0.9% of the population requires intermittent or continuous
treatment with steroids. Up to 2.5% of the elderly population (age >70
years) take corticosteroids. In addition to the risks imposed by steroids,
some of the underlying conditions requiring steroids also predispose to
osteoporosis (e.g. rheumatoid arthritis and multiple myeloma). The
pathogenesis of glucocorticoid-induced osteoporosis is complex,
occurring in two phases with rapid bone loss of up to 12% of total BMD
within the first year followed by a slower rate of bone loss of typically up
to 3% per year. Corticosteroids decrease the formation of osteoblasts and
markedly increase (by up to three-fold) apoptosis of osteoblasts. The
early phase with increased bone loss is also characterized by increased
osteoclastic activity. Decreased osteoprotegerin (OPG) may be partly
responsible for this effect. OPG acts as a soluble decoy receptor for the
receptor activator of nuclear factor-k-beta ligand (RANKL), and
decreased activity of OPG allows increased interaction between RANKL
and its native receptor (RANK) leading to activation of the pathway that
includes nuclear factor-k-beta (NF-kB). Other mechanisms involved
include decreased expression of bone morphogenetic proteins and a
diversion of bone marrow precursor cells into the adipocyte lineag
Case History
SP is a 45-year-old woman who has suffered asthma since childhood. The
asthma is now stable, but she is concerned about osteoporosis since she
has had an average of three courses of steroids each year for the past ten
years. She has regular periods but she is approaching the menopause. Her
mother developed osteoporosis in later life. A dual energy X-ray
absorptiometry (DEXA) scan confirms that she has low bone mineral
density (BMD).
1.What are the causes of steroid-induced osteoporosis?
2.How might the condition be prevented?
3.What treatment options are available?
4.what are new concepts
Problem 22
• Most likely diagnosis: Streptococcal pharyngitis.
Dangerous causes of sore throat: Epiglottitis,
peritonsillar abscess, retropharyngeal abscess,
Ludwig angina.
Objectives
1. Recognize the different etiologies of pharyngitis, paying close attention
to those that are potentially life-threatening.
2. Be familiar with widely accepted decision-making strategies for the
diagnosis and management of group A β-hemolytic streptococcal
(GABS) pharyngitis.
3. Learn the treatment of GABS pharyngitis and understand the sequelae
of this disease.
4. Recognize acute airway emergencies associated with upper airway
infections
Case history
A 13-year-old adolescent boy presents to the emergency department with
a chief complaint of sore throat and fever for 2 days. He reports that his
younger sister has been ill for the past week with “the same thing.” The
patient has pain with swallowing, but no change in voice, drooling, or
neck stiffness. He denies any recent history of cough, rash, nausea,
vomiting, or diarrhea. He denies any recent travel and has completed the
full series of childhood immunizations. He has no other medical
problems, takes no medications, and has no allergies.
On examination, the patient has a temperature of 38.5°C (101.3°F), a
heart rate of 104 beats per minute, blood pressure 110/60 mm Hg, a
respiratory rate of 18 breaths per minute, and an oxygen saturation of
99% on room air. His posterior oropharynx reveals erythema with
tonsillar exudates without uvular deviation, or significant tonsillar
swelling. Neck examination is supple without tenderness of the anterior
lymph nodes. Chest and cardiovascular examination is unremarkable. His
abdomen is soft and not tender with normal bowel sounds and no HSM ,
Skin is without rash.
1 . What is the most likely diagnosis?
2 . What are the dangerous causes of sore throat you don’t want to miss?
3 . What is your diagnostic plan?
4 . What is your therapeutic plan?
5 . The patient after 5 days of his illness develops Sudden onset of fever,
drooling, tachypnea, stridor, toxic appearing and on Lateral cervical
radiograph there is a (thumb-printing sign) What is this complication and
how are you going to manage ?
Problem 23
Pneumonia
Background
Pneumonia is an inflammatory condition of the lung affecting primarily
the alveoli. It is usually caused by infection with viruses or bacteria and
less commonly other microorganisms, certain drugs and other conditions
such as autoimmune diseases.
Typical symptoms include a cough, chest pain, fever, and difficulty
breathing. Diagnostic tools include x-rays and culture of the
sputum. Vaccines to prevent certain types of pneumonia are available.
Treatment depends on the underlying cause. Pneumonia presumed to be
bacterial is treated with antibiotics. If the pneumonia is severe, the
affected person is, in general, admitted to hospital.
Pneumonia affects approximately 450 million people globally per year,
seven percent of population, and results in about 4 million deaths, mostly
in third-world countries. Although pneumonia was regarded by William
Osler in the 19th century as "the captain of the men of death" the advent
of antibiotic therapy and vaccines in the 20th century has seen
improvements in survival. Nevertheless, in developing countries, and
among the very old, the very young, and the chronically ill, pneumonia
remains a leading cause of death. In the terminally ill and elderly,
especially those with other conditions, pneumonia is often the immediate
cause of death.
Objectives
Upon completion of this case, the student should be able to:
1. Analyze the development and classification of pneumonia
infection.
2. Evaluate the epidemiology and risk factors of community-acquired
pneumonia.
3. Identify etiological factors of community-acquired pneumonia,
including pathogens most commonly associated with infection in
adults and children.
4. Use established diagnostic criteria to appropriately identify and
categorize community-acquired pneumonia.
5. Outline the management of community-acquired pneumonia in
adults and children, including the use of antibiotic therapy and
appropriate site of care.
6. Discuss prevention strategies, including vaccination, for
community-acquired pneumonia.
Case study
A 68 year old man presented with a harsh, productive cough four days
prior to admission. The sputum is thick and yellow with streaks of blood.
He developed a fever, shaking, chills and malaise along with the cough.
One day ago he developed pain in his right chest that intensifies with
inspiration. Past history reveals that he had a chronic smoker's cough for
"10 or 15 years" which he describes as being mild, non-productive and
occurring most often in the early morning. He smoked 2 packs of
cigarettes per day for the past 50 years. The patient is a retired truck
driver who has been treated for mild hypertension, bronchitis,
appendicitis (as a young adult), hemorrhoids and a fractured femur and
splenic injury. (motorcycle accident).
PHYSICAL EXAMINATION: The patient is an elderly man who
appears tired and underweight. He coughs continuously. Sitting in a chair,
he leans to his right side, holding his right chest with his left arm. Vital
signs are as follows: blood pressure 152/90, apical heart rate 112/minute
and regular, respiratory rate 24/minute and temperature 38.8 c. Both
lungs are resonant by percussion with one exception: the right midanterior and right mid-lateral lung fields are dull. Auscultation reveals
bilateral diminished vesicular breath sounds. Bronchial breath
sounds, rhonchi and late inspiratory crackles (are heard) in the area of the
right mid-anterior and right mid-lateral lung fields. The remainder of the
lung fields is clear. Percussion and auscultation of the heart reveals no
significant abnormality.
LABORATORY: WBC 17,000/mm3; neutrophils 70%, bands 15%,
lymphocytes 15%.
1.Identify the problems from the history.
2. Identify and explain the significance of physical findings.
3. Review the lab findings. What is your diagnosis?
4. What do you understand by the terms "hospital acquired" and
"community acquired " pneumonia.? Which type of pneumonia does our
patient have?
5. What organisms are likely to be causing his pneumonia?
6. List the various host factors, or conditions which predispose a patient
to developing pneumonia. What host factors may have predisposed this
patient to pneumonia?
7. Explain the pathogenesis of pneumococcal pneumonia? What virulence
factors are important? What pathologic changes are produced in the lungs
because of pneumonia?
8. How is the specific diagnosis established? What is the primary
disadvantage to the examination of expectorated sputum? Describe
characteristic morphology/growth of S. pneumoniae.
9. What antimicrobial agents would you prescribe for this patient? Would
you use or avoid penicillin, and why? What is the duration of treatment?
10. What is the mechanism of pneumococcal resistance to penicillin?
11. What are the complications of Pneumococcal pneumonia?
12. Is prevention possible?
Problem24
Asthma
Background
Asthma is a chronic inflammatory respiratory disorder causes recurrent
episodes
of wheezing, breathlessness, chest tightness and cough, especially at night
or in the early morning. These asthma episodes are associated with
airflow limitation or obstruction that is reversible either spontaneously or
with treatment. Asthma usually begins in childhood or adolescence, but it
also may first appear during adult years. While the symptoms may be
similar, certain important aspects of asthma are different in children and
adults. Bronchial asthma is the more correct name for the common form
of asthma. The term 'bronchial' is used to differentiate it from 'cardiac'
asthma, which is a separate condition that is caused by heart failure.
Although the two types of asthma
havesimilar symptoms, including wheezing (a whistling sound in the ches
t) andshortness of breath, they have quite different causes.
Bronchial asthma is usually intrinsic (no cause can be demonstrated), but
is occasionally caused by a specific allergy (such as allergy to mold,
dander, dust).
Objectives
Upon completion of this case, the student should be able to:
 Identify the signs and symptoms that can be used to assess an
asthma attack.
 Explain the underlying disease process of asthma.
 Describe treatment of an asthma attack
 Discuss the systemic approach to the treatment of an asthma attack.
 Identify common asthma medications, including beta-2 agonists
and inhaled corticosteroids.
Case Study
A 37 y/o female with a history of asthma, presents to the ER with
tachypnea, and acute shortness of breath with audible wheezing. Patient
has taken her prescribed medications of Cromolyn Sodium and Ventolin
at home with no relief of symptoms prior to coming to the ER. A physical
exam revealed the following: HR 110, RR 40 with signs of accessory
muscle use. Ausculation revealed decreased breath sounds with
inspiratory and expiratory wheezing and pt was coughing up small
amounts of white sputum. SaO2 was 93% on room air. An arterial blood
gas (ABG) was ordered with the following results: pH 7.5, PaCO2 27,
PaO2 75. An aerosol treatment was ordered and given with 0.5 cc
albuterol with 3.0 cc normal saline in a small volume nebulizer for 10
minutes. Peak flows done before and after the treatment were 125/250
and ausculation revealed loud expiratory wheezing and better airflow. 20
minutes later a second treatment was given with the above meds. Peak
flows before and after showed improvements of 230/360 and on
ausculation there was clearing of breath sounds and much improved
airflow. RR was 24 at this time and HR 108. Symptoms resolved and
patient was given prescription for inhaled steroids to be used with current
home meds. Instruction was given for use of inhaled steroids and the
patient was sent home.
1. list different ways asthma is diagnosed.
2. explain how a proper forced expiratory test is to be performed.
3. list the 3 primary pathologic reactions during as asthmatic episode.
4. differentiate the two types of asthma.
5. recognize chest x-ray changes seen with asthma.
6. recognize the signs and symptoms of asthma.
7. discuss which values are most often used when testing for asthma.
8. recommend the appropriate drug therapy of an asthmatic patient for
chronic treatment or for an acute attack.
9. recognize the commonly used bronchodilation agents.
10.list the side effects related to the different asthma medications.
Problem25
Epilepsy
Background
Seizure is defined as temporary, self-limited cerebral dysfunction as a
result of abnormal, self-limited hypersynchronous electrical discharge of
cortical neurons.
There are many kinds of seizures, each with characteristic behavioral
changes and each usually with particular EEG recordings.
Essentially, seizures are considered to relate to only one of the two
cerebral hemispheres (these are referred to as partial or focal seizures) or
both hemispheres
of the brain (generalized seizures). Where the seizure pattern initiates and
where it spreads determines the type of seizure, relates to prognosis, and
frequently warrants different therapies.
Partial seizures are further subdivided depending on whether the patient
has a change in level of (or loses) consciousness. In simple partial
seizures, patients do not lose consciousness, but in complex partial
seizures they do have an alteration or loss of consciousness.
Subcategorization of generalized seizures primarily reflects the type of
motor disturbances present during the convulsion (e.g., tonic-clonic,
tonic, atonic, myoclonus).
Seizure disorders can present as intermittent events. The initial event,
whether
reported by the patient or witnessed by an observer, is often clinically
reliable
as to whether a seizure begins with a focal onset or is immediately
generalized.
Approximately 10–25% of patients who complain of a seizure without an
obvious cause see a physician after having only one seizure. This is
usually a tonic-clonic event and most have no risk factors for epilepsy.
These patients usually have a normal neurologic examination, normal
EEG, and normal radiographic testing. Of these patients, one-quarter will
prove to have epilepsy.
Although one needs to know whether he had any predisposing factors to
seizures, such as increased alcohol consumption, drugs that can lower
seizure threshold (e.g.,cocaine, amphetamines), or sleep deprivation.
Many neurologists wait until the second seizure before initiating
treatment.
The physician should discuss with the patient (or parents or both) the
implications of treating or not treating.
Objectives
1. Know diagnostic approach to first seizure in an adult, including the
importance of history, examination, and testing.
2.Memorise the classification of seizures.
3. Describe the workup and follow-up for the patient.
4.Understanding the precipitating facters.
5.Know the indications of antiepileptic therapy and its side effects
Case history
A 23-year-old graduate student was studying late at night for an
examination.
He recalls studying, but his next memory is being on the floor and aching
throughout his body. He was incontinent of urine but not stool and felt
slightly confused. No one was with him, and he did not know what to do.
He called his mother, who recommended he go to the local emergency
room.
In the emergency room, he suddenly fell to the ground unconscious with
muscle stiffening, tonic body posturing and eyes uprolling. He became
cyanosed, passed frothy mouth discharge with tinge of blood, and
produced a shrill of crying. After about 2 minutes he developed
rhythmical jerky limbs movement which faded gradually over further 2
minutes to end by passing urine. He became confused, pointed to his
head, and vomited twice. Then he slept for half an hour to awake well but
he couldn’t memorise the attack.
Case 2
A 40 years old lady consulted a neurologist for frequent episodes of brief
lapses of consciousness followed by postictal drowsiness and fatigue.
They might occur on weekly bases, as maximum as 5 times a day and as
minimum as once a day. A witness admitted that she stared blankly and
produced involuntary orolabial movements (snout and lip smacking) and
forceful shaking of both of her hands.
◆ What is the most likely diagnosis?
◆ What is the next diagnostic step?
◆ What is the next step in therapy?
.
Problem 26
Coma
Background
Coma: refers to a state of unresponsiveness whereby a patient is
incapable of purposefully responding to the most vigorous and noxious
stimuli.
Consciousness depends on an intact and interacting brainstem reticular
formation and cerebral hemispheres.
While diffuse brain injuries and dysfunction affect the reticular formation
in an intuitively obvious manner, focal injuries and dysfunction leading to
impaired consciousness are more complex:
• A focal injury to the upper brainstem will adversely affect reticular
formation function at its origin.
• Cerebellar mass lesions may cause obstructive hydrocephalus leading to
transtentorial herniation and upper brainstem compression or ischemia
from vascular torsion. Direct brainstem compression, upward herniation
through the tentorial notch, and downward herniation through the
foramen
magnum from cerebellar mass lesions may also occur.
• Hemispheric lesions must be multiple and bilateral to cause reticular
formation dysfunction in both hemispheres, or have substantial mass
effect to cause remote
injury to the reticular formation within the contralateral hemisphere or
brainstem.
Standardized scoring systems such as the Glasgow Coma Scale,
originally developed and validated for trauma patients, may have clinical
and prognostic value in patients with selected etiologies of impaired
consciousness.
Disorders of consciousness can be categorized in different ways.
Although helpful for considering a differential diagnosis, these
classification schemes may have considerable overlap. Etiologies can be
categorized as:
• Diffuse or focal.
• Toxic or metabolic, structural, and/or functional.
• Transient and brief, protracted but reversible, or
permanent.
Clinical examination, imaging, and electroencephalography (EEG) are
essential in differentiating between these. Diffuse etiologies of impaired
consciousness account for up to 70% of all disorders of consciousness.
While assessing a comatose patient, the diagnostic evaluation and
management should occur simultaneously.
Subjects with a Glasgow Coma Scale score of less than 8 are probably
unable to protect their airway and should be intubated.
The prognosis for patients with impaired consciousness ranges from full
recovery to death, and depends on the etiology, duration, and treatment of
impaired consciousness and the presence of comorbid disease states.
Objectives
1-Know the definition of consciousness, its anatomy and physiology.
2-Know the definition of COMA and levels of loss of consciousness
3-Understanding GLASGOW COMA SCALE.
4-Know mimickers of coma
5-Approaching comatosed patients
6-management of comatose patients
Case 1
A 40 years woman with 3 days history of occipital headache, morning
vomiting and low grade fever; was brought to the emergency room with
complete loss of responsiveness. By examination, she was comatosed,
having positive neck stiffness, symmetrical reactive pupils, intact
oculocephalic reflex.
Her temperature was 39.5 C.
Case 2
A 55 year old alcoholic man was brought to the emergency room with 2
hours deterioration of consciousness. On exam he was comatosed,
jaundiced, acidotic breathing with musty fetor oris. There was clear
abdominal distension.
1-What is the likely cause of coma?
2-What are the key points for the diagnosis?
3- Mention five important investigations.
4-Mention your management plan.
Problem 27
Peptic Ulcer
Background:
DYSPEPSIA: Pain or discomfort centered in the upper abdomen (mainly
in or
around the midline), which can be associated with fullness, early satiety,
bloating,or nausea. Dyspepsia can be intermittent or continuous, and it
may or may not be related to meals.
FUNCTIONAL (NONULCER) DYSPEPSIA: Symptoms as described
for dyspepsia,persisting for at least 12 weeks but without evidence of
ulcer on endoscopy.
HELICOBACTER PYLORI: A gram-negative microaerophilic bacillus
that resides within the mucus layer of the gastric mucosa and causes
persistent gastric infection and chronic inflammation. It produces a urease
enzyme that splits urea, raising local pH and allowing it to survive in the
acidic environment. H pylori is associated with 50% to 60% of gastric
ulcers and with 70% to 90% of duodenal ulcers.
PEPTIC ULCER DISEASE (PUD): Peptic ulcer disease represents a
serious medical problem. Approximately 500,000 new cases are reported
each year, with 5 millionpeople affected in the United States alone.
Interestingly, those at the highest risk of contracting peptic ulcer disease
are those generations born around the middle of the 20th century. Ulcer
disease has become a disease predominantly affecting the older
population, with the peak incidence occurring between 55 and 65 years of
age. In men, duodenal ulcers were more common than gastric ulcers; in
women, the converse was found to be true. Thirty-five percent of patients
diagnosed with gastric ulcers will suffer serious complications. Although
mortality rates from peptic ulcer disease are low, the high prevalence and
the resulting pain, suffering, and expense are very costly. Ulcers can
develop in the esophagus, stomach or duodenum, at the margin of a
gastroenterostomy, in the jejunum, in Zollinger-Ellison syndrome, and in
association with a Meckel's diverticulum containing ectopic gastric
mucosa. Peptic ulcer disease is one of several disorders of the upper
gastrointestinal tract that is caused, at least partially, by gastric acid.
Patients with peptic ulcer disease may present with a range of symptoms,
from mild abdominal discomfort to catastrophic perforation and bleeding
CLINICAL APPROACH
Upper abdominal pain is one of the most common complaints
encountered in primary care practice. Many patients have benign
functional disorders (ie, no specific pathology can be identified after
diagnostic testing), but others have potentially more serious conditions
such as PUD or gastric cancer. Historical clues, knowledge of the
epidemiology of diseases, and some simple laboratory assessments can
help to separate benign from serious causes of pain. However, endoscopy
is often necessary to confirm the diagnosis. Dyspepsia refers to upper
abdominal pain or discomfort that can be caused by PUD, but it also can
be produced by a number of other gastrointestinal disorders.
Gastroesophageal reflux typically produces “heartburn,” or burning
epigastric or mid chest pain, usually occurring after meals and worsening
with recumbency. Biliary colic caused by gallstones typically has acute
onset of severe pain located in the right-upper quadrant or epigastrium,
usually is precipitated by meals, especially fatty foods, lasts 30 to 60
minutes with spontaneous resolution, and is more common in women.
Irritable bowel syndrome is a diagnosis of exclusion but is suggested by
chronic dysmotility symptoms (bloating, cramping) often relieved with
defecation, sometimes alternating constipation and diarrhea, without
weight loss or GI bleeding. If these causes are excluded by history or
other investigations, it is still difficult to clinically distinguish the patients
with PUD from those without ulcers, termed nonulcer dyspepsia. The
classic symptoms of duodenal ulcers are caused by the presence of acid
without food or other buffers. Symptoms are typically produced after the
stomach is emptied but food-stimulated acid production still persists,
typically 2 to 5 hours after a meal. They may awaken patients at night,
when circadian rhythms increase acid production. The pain is typically
relieved within minutes by neutralization of acid by food or antacids (eg,
calcium carbonate, aluminum-magnesium hydroxide).Gastric ulcers, by
contrast, are more variable in their presentation. Food may actually
worsen symptoms in patients with gastric ulcer, or pain might not be
relieved by antacids. In fact, many patients with gastric ulcers have no
symptoms at all. Five percent to 10% of gastric ulcers are malignant, and
so should be investigated endoscopically and biopsied to exclude
malignancy. Gastric cancers may present with pain symptoms, with
dysphagia if they are located in the cardiac region of the stomach, with
persistent vomiting if they block the pyloric channel, or with early satiety
by their mass effect or infiltration of the stomach wall. Because the
incidence of gastric cancer increases with age, patients older than 45
years who present with new-onset dyspepsia should generally undergo
endoscopy. In addition, patients with alarm symptoms (eg, weight loss,
recurrent vomiting, dysphagia, evidence of GI bleeding, or irondeficiency anemia) should be referred for prompt endoscopy. Finally,
endoscopy should be recommended for patients whose symptoms have
failed to respond to empiric therapy. When endoscopy is undertaken,
besides visualization of the ulcer, biopsy samples can be taken to exclude
the possibility of malignancy, and specimens can be obtained for urease
testing or microscopic examination to prove current H pylori infection. In
younger patients with no alarm features, an acceptable strategy is to
perform a noninvasive test to detect H pylori, such as serology, urea
breath test, or fecal H pylori (Hp) antigen test. The two most commonly
used tests are the urea breath test, which provides evidence of current
active infection, and H pylori antibody tests, which provide evidence of
prior infection, but will remain positive for life,even after successful
treatment. Because chronic infection with H pylori is found in the large
majority of duodenal and gastric ulcers, the standard of care is to test for
infection and, if present, to treat it with a combination antibiotic regimen
for 14 days and acid suppression with a proton-pump inhibitor or H2blocker. Several different regimens are used, such as omeprazole plus
clarithromycin, plus metronidazole or amoxicillin. A bismuth compound
such as bismuth subsalicylate is also frequently included. To improve
patient compliance, some anti-H pylori regimens are available in
prepackaged formulations. Aside from its association with PUD, H pylori
is associated with the development of gastric carcinoma and gastric
mucosa–associated lymphoid tissue (MALT) lymphoma. Whether
treatment of H pylori infection reduces or eliminates dyspeptic symptoms
in the absence of ulcers (nonulcer dyspepsia) is uncertain. Similarly,
whether treatment of asymptomatic patients found to be H pylori positive
is beneficial is unclear. In H pylori–positive patients with dyspepsia,
antibiotic treatment may be considered, but a follow-up visit is
recommended within 4 to 8 weeks. If symptoms persist or alarm features
develop, then prompt upper endoscopy is indicated. In addition to H
pylori, the other major cause of duodenal and gastric ulcers is the use of
NSAIDs. They promote ulcer formation by inhibiting gastroduodenal
prostaglandin synthesis, resulting in reduced secretion of mucus and
bicarbonate and decreased mucosal blood flow. In other words, they
impair local defenses against acid damage. The risk of ulcer formation
caused by NSAID use is dose dependent and can occur within days after
treatment is initiated. If ulceration occurs, the NSAID should be
discontinued if possible, and acid-suppression therapy with a protonpump inhibitor should be initiated.A rare cause of ulcer is the ZollingerEllison syndrome (ZES), a condition in which a gastrin-producing tumor
(usually pancreatic) causes acid hypersecretion, peptic ulceration, and
often diarrhea. This condition should be suspected if patients have ulcers
refractory to standard medical therapy, ulcers in unusual locations
(beyond the duodenal bulb), or ulcers without a history of NSAID use or
H pylori infection. About 25% of gastrinomas occur in patients with
multiple endocrine neoplasia I (MEN I) syndrome, an autosomal
dominant genetic disorder characterized by parathyroid, pancreatic, and
pituitary neoplasms. To diagnose Zollinger-Ellison syndrome, the first
step
is to measure a fasting gastrin level, which may be markedly elevated
(>1000 pg/mL), and then try to localize the tumor with an imaging study.
Hemorrhage is the most common severe complication of PUD and can
present
with hematemesis or melena. Free perforation into the abdominal cavity
may
occur in association with hemorrhage, with sudden onset of pain and
development of peritonitis. If the perforation occurs adjacent to the
pancreas, it may induce pancreatitis. Some patients with chronic ulcers
later develop gastric outlet obstruction, with persistent vomiting and
weight loss but no abdominal distention. Perforation and obstruction are
indications for the patient to undergo surgery.
In conclusion :
 The most common causes of duodenal and gastric ulcers are
Helicobacter pylori infection and use of nonsteroidal antiinflammatory drugs.
 Helicobacter pylori is associated with duodenal and gastric ulcers,
chronic active gastritis, gastric adenocarcinoma, and gastric
mucosa–associated lymphoid tissue (MALT) lymphoma.
 Treatment of peptic ulcers requires acid suppression with an H2
blocker or proton-pump inhibitor to heal the ulcer, as well as
antibiotic therapy of Helicobacter pylori infection, if present, to
prevent recurrence.
 Patients with dyspepsia who have “red flag” symptoms (new
dyspepsia after the age of 45 years, weight loss, dysphagia,
evidence of bleeding or anemia) should be referred for an early
endoscopic examination.
 Other patients (patients with dyspepsia who do not have “red flag”
symptoms) may be tested for Helicobacter pylori and treated first.
Antibody tests show evidence of infection but remain positive for
life, even after successful treatment. Urea breath tests are evidence
of current infection.
 Common treatment regimens for Helicobacter pylori infection
include a 14-day course of a proton-pump inhibitor in high doses
along with antibiotic therapy, which may include clarithromycin,
amoxicillin, metronidazole, or tetracycline, along with a bismuth
compound.
Objectives
1. Know how to differentiate common causes of abdominal pain by
historical clues.
2. Recognize clinical features of duodenal ulcer, gastric ulcer, and
features that
increase concern for gastric cancer.
3. Understand the role of H pylori infection and use of nonsteroidal antiinflammatory drugs (NSAIDs) in the etiology of PUD.
4. Understand the use and interpretation of tests for H pylor
Case history
A 37-year-old man presents with complaints of chronic and recurrent
upper abdominal pain which awake the patient from his sleep , the pain is
burning, occurs when the stomach is empty, and is relieved within
minutes by food or antacids. He does not have evidence of
gastrointestinal bleeding or anemia. He does not take nonsteroidal antiinflammatory drugs, but he does have serologic evidence of H pylori
infection.O/E he has normal vital signs and he uses his index figure to
point to the site of pain at the epigastric region .
1 . What is the most likely diagnosis: Peptic ulcer disease (PUD).
2 .How could you make the diagnosis ?
3 . What is the Next step in the management of this patient ? : Triple
antibiotic therapy for H pylori infection, and acid suppression.
4 . What are the important complications might develop in this patient ?
5 . If the patient develops weight loss of about 7 kilos within 6 months
period what you expect the causes ?
.
Considerations
In this patient, the symptoms are suggestive of duodenal ulcer. He does
not have “alarm symptoms,” such as weight loss, bleeding, or anemia,
and his young age and chronicity of symptoms make gastric malignancy
an unlikely cause for his symptoms. H pylori commonly is associated
with PUD and requires eradication to promote ulcer healing and prevent
recurrence. This patient’s symptoms might also represent nonulcer
dyspepsia
REFERENCES
Bytzer P, Talley NJ. Dyspepsia. Ann Intern Med. 2001;134:815.
Del Valle J. Peptic ulcer disease and related disorders. In: Longo DL, Fauci AS,
Kasper, DL, et al., eds.
Harrison’s Principles of Internal Medicine. 18th ed. New York, NY: McGrawHill; 2012:2438-2460.
Suerbaum S, Michetti P. Medical progress: Helicobacter pylori infection. N Engl
J Med. 2002;347: 1175-1186.
Davidson's Principle & Practice of Medicine . 22nd Ed . London; Churchil
Livingstone ; 2014: 871 - 878
Problem 28
Acute lymphoblastic leukemia
Background
Acute lymphoblastic leukemia (ALL) or acute lymphoid leukemia is
an acute form of leukemia, or cancer of the white blood cells,
characterized by the overproduction of cancerous, immature white blood
cells—known as lymphoblasts In persons with ALL, lymphoblasts are
overproduced in the bone marrow and continuously multiply, causing
damage and death by inhibiting the production of normal cells—such
as red and white blood cells and platelets—in the bone marrow and by
spreading (infiltrating) to other organs. ALL is most common in
childhood with a peak incidence at 2–10 years of age, and another peak in
old age
The symptoms of ALL are indicative of a reduced production of
functional blood cells, because the leukemia wastes the resources of
the bone marrow, which are normally used to produce new, functioning
blood cells.These symptoms can include fever, increased risk of infection
(especially bacterial infections like pneumonia, due to neutropenia;
symptoms of such an infection include shortness of breath, chest pain,
cough, vomiting, changes in bowel or bladder habits), increased tendency
to bleed (due to thrombocytopenia) and signs indicative
of anemia including pallor, tachycardia (high heart rate), fatigue and
headache
Signs and symptoms
Initial symptoms are not specific to ALL, but worsen to the point that
medical help is sought. They result from the lack of normal and healthy
blood cells because they are crowded out by malignant and immature
leukocytes (white blood cells). Therefore, people with ALL experience
symptoms from malfunctioning of their erythrocytes (red blood cells),
leukocytes, and platelets. Laboratory tests that might show abnormalities
include blood count tests, renal function tests, electrolyte tests, and liver
enzyme tests.
The signs and symptoms of ALL are variable but follow from bone
marrow replacement and/or organ infiltration.
 Generalized weakness and fatigue
 Anemia
 Dizziness
 Frequent or unexplained fever and infection
 Weight loss and/or loss of appetite

Excessive and unexplained bruising
 Bone pain, joint pain (caused by the spread of "blast" cells to the
surface of the bone or into the joint from the marrow cavity)
 Breathlessness
 Enlarged lymph nodes, liver and/or spleen
 Pitting edema (swelling) in the lower limbs and/or abdomen
 Petechiae, which are tiny red spots or lines in the skin due to
low platelet levels
Diagnosis
Diagnosing ALL begins with a medical history, physical
examination, complete blood count, and blood smears. Because the
symptoms are so general, many other diseases with similar symptoms
must be excluded. Typically, the higher the white blood cell count the
worse the prognosis ]Blast cells are seen on blood smear in majority of
cases (blast cells are precursors (stem cells) to all immune cell lines).
A bone marrow biopsy is conclusive proof of ALL. A lumbar
puncture (also known as a spinal tap) will tell if the spinal column
and brain have been invaded.
Pathological examination, cytogenetics (in particular the presence
of Philadelphia chromosome), and immunophenotyping establish whether
myeloblastic (neutrophils, eosinophils, or basophils) or lymphoblastic (B
lymphocytes or T lymphocytes) cells are the problem. RNA testing can
establish how aggressive the disease is; different mutations have been
associated with shorter or longer
survival. Immunohistochemicaltesting may
reveal TdT or CALLA antigens on the surface of leukemic cells. TdT is a
protein expressed early in the development of pre-T and pre-B cells,
whereas CALLA is an antigen found in 80% of ALL cases and also in the
"blast crisis" of CML.
Medical imaging (such as ultrasound or CT scanning) can find invasion
of other organs commonly the lung, liver, spleen, lymph nodes, brain,
kidneys, and reproductive organs.[4]
Subtyping of the various forms of ALL used to be done according to
the French-American-British (FAB) classification, which was used for all
acute leukemias (including acute myelogenous leukemia, AML).
 ALL-L1: small uniform cells
 ALL-L2: large varied cells
 ALL-L3: large varied cells with vacuoles (bubble-like features)
Each subtype is then further classified by determining the surface markers
of the abnormal lymphocytes, called immunophenotyping. There are 2
main immunologic types: pre-B cell and pre-T cell. The mature B-cell
ALL (L3) is now classified as Burkitt's lymphoma/leukemia. Subtyping
helps determine the prognosis and most appropriate treatment in treating
ALL
Treatment
The earlier acute lymphocytic leukemia is detected, the more effective the
treatment. The aim is to induce a lasting remission, defined as the absence
of detectable cancer cells in the body (usually less than 5% blast cells in
the bone marrow).
Treatment for acute leukemia can
include chemotherapy, steroids, radiation therapy, intensive combined
treatments (including bone marrow or stem cell transplants), and growth
factors.
Chemotherapy
Chemotherapy is the initial treatment of choice. Most ALL patients will
receive a combination of different treatments. There are no surgical
options, due to the body-wide distribution of the malignant cells. In
general, cytotoxic chemotherapy for ALL combines multiple
antileukemic drugs in various combinations. Chemotherapy for ALL
consists of three phases: remission induction, intensification, and
maintenance therapy.
Case history
a 10 -year-old boy, brought by his mother into the ER because of a highgrade fever. She has tried tylenol (acetaminophen), but the fever is "just
not going away". The child has been very irritable for the last 4 days and
is crying a lot. He is also pulling his ear and not eating well. The child
has been generally well other than occasional sore throat, this season. His
vitals are, Temperature: 38.8C(102.2F); BP: 90/60 mm Hg; PR: 119/min;
RR: 24/min. Physical examination revealed a well nourished, but irritable
child. He had enlarged cervical lymph nodes and splenomegaly. The
tympanic membranes were inflammed. A stat CBC is ordered which
shows
WBC
81,100mm3
Hemoglobin 8.0
Hematocrit 27%,
Platelets
16,000mm3
The leukocyte distribution on differential smear was
Blast forms
80%
Prolymphocytes 10%
Lymphocytes 10%
The blast cells had one or two nuclei and no Auer rods were seen.
Subsequent cytochemical reaction reveals strongly positive periodic acid
schiff reaction. What is the most probable diagnosis?
A. Burkitt’s lymphoma
B. Acute myelocytic leukemia
C. Prolymphocytic leukemia
D. Acute lymphoblastic leukemia
E. Myelodysplastic syndrome
Explanation:
The predominant type of leukemia in children from ages 2-10 years is
acute lymphoblastic leukemia (ALL). It presents acutely with child
feeling ill for just a couple of weeks. About 30-50% of them present with
infections. Lymphadenopathy and splenomegaly occur in about half.
ALL presents with lymphoblasts in the peripheral smear. The patients
have varying degree of anemia, neutropenia, and thrombocytopenia.
Presence of more than 25% lymphoblasts in bone marrow and the
positive periodic acid schiff reaction (PAS) staining makes the diagnosis
of ALL. Good prognostic indications are youth and hyperploidy while
poor prognosis is indicated by increasing age.
Lymphoma is the preferred term when the process is confined to a mass
lesion with minimal or no blood and marrow involvement. Burkitt’s
lymphoma is a diffuse, small, undifferentiated high-grade malignancy
and is associated with Epstein-Barr virus. A starry sky appearance of the
tumor cells is described with it.
In acute myelocytic leukemias the predominant cells are of myeloid
origin. Auer rods are specific for AML.
Prolymphocytic leukemia is a CLL variant with massive splenomegaly,
high lymphocytosis with little lymphadenopathy.
Myelodysplastic syndromes (MDS) are clonal stem cell disorders, which
may progress to acute leukemias. They are usually seen in the adults and
characterized by pancytopenia.
Educational Objective:
Acute lymphoblastic leukemia is predominantly a disease of children and
recognize the importance of periodic acid schiff positive staining in
diagnosis.
Problem 29
Heart Failure
Objective:
1234567-
To know the presentation of patient with decompensated HF
To know the clinical feature of patient with HF
To understand the pathophysiology of HF
To know the precipitating factors that decompensate the HF
To know the investigation that need to the patient with HF
To know the treatment that improve the prognosis of HF
To know the treatment that just improve the symptoms of the
patient with HF
Case History
65 years old male patient with past history of IHD ,present to ED with
SOB at lying flat ,with palpitation o examination patient was irritable and
had tachypnea ,tachycardia , Bpr was 160/90 ,bilateral leg edema ,and 7
mm raised of JVP. On respiratory examination patient had 15 mm
distance bilateral fine basal crackle with CVS examination show S3, S4
gallop rhythm. What’s yours :
abcd-
provisional Dx
investigation that send to the patient
immediate treatment
definitive treatment
Background
Heart failure describes the clinical syndrome that develops when the heart
cannot maintain an adequate cardiac output, or can do so only at the
expense of an elevated filling pressure. In mild to moderate forms of
heart failure, cardiac output is adequate at rest and only becomes
inadequate when the metabolic demand increases during exercise or some
other form of stress.
Causes of heart failure
1-Reduced ventricular contractility
2-Ventricular outflow Obstruction (pressure overload)
3-Ventricular inflow obstruction
4-Ventricular volume overload
5-Arrhythmia
6-Diastolic dysfunction
a-Constrictive pericarditis
b-Restrictive cardiomyopathy
c-Left ventricular hypertrophy and fibrosis
d-Cardiac tamponade
Pathophysiology:
Factors that precipitate heart failure:
1-Myocardial ischaemia or infarction
2- Intercurrent illness, e.g. infection
3- Arrhythmia, e.g. atrial fibrillation
4-Inappropriate reduction of therapy
5-Administration of a drug with negative inotropic properties (e.g. βblocker) or fluid-retaining properties (e.g. nonsteroidal anti-inflammatory
drugs (NSAIDs), corticosteroids)
6- Pulmonary embolism
7-Conditions associated with increased metabolic demand, e.g.
pregnancy, thyrotoxicosis, anaemia
8- I.v. fluid overload, e.g. post-operative i.v. infusion
Clinical feature:
Investigations
Chest X-ray
Management of acute pulmonary oedema
This is urgent:
• Sit the patient up in order to reduce pulmonary congestion.
• Give oxygen (high-flow, high-concentration). Noninvasive positive
pressure ventilation (continuous positive airways pressure (CPAP) of 5–
10 mmHg) by a tight-fitting facemask results in a more rapid
improvement in the patient’s clinical state.
• Administer nitrates, such as i.v. glyceryl trinitrate 10– 200 g/min or
buccal glyceryl trinitrate 2–5 mg, titrated upwards every 10 minutes, until
clinical improvement occurs or systolic BP falls to < 110 mmHg.
• Administer a loop diuretic such as furosemide 50– 100 mg i.v.
The patient should initially be kept on strict bed rest with continuous
monitoring of cardiac rhythm, BP and pulse oximetry. Intravenous
opiates may be cautiously used when patients are in extremis. They
reduce sympathetically mediated peripheral vasoconstriction but may
cause respiratory depression and exacerbation of hypoxaemia and
hypercapnia.
If these measures prove ineffective, inotropic agents may be required to
augment cardiac output, particularly in hypotensive patients. Insertion of
an intra-aortic balloon pump can be very beneficial in patients with acute
cardiogenic pulmonary oedema, especially when secondary to myocardial
ischaemia.
Management of chronic heart failure
General measures
Education of patients and their relatives about the causes and treatment of
heart failure can help adherence to a management plan . Some patients
may need to weigh themselves daily and adjust their diuretic therapy
accordingly.
Drug therapy
1-Diuretic therapy: reduces preload, not improve mortality
2-Vasodilator therapy: as nitrates, reduce preload, and arterial dilators,
such as hydralazine, reduce afterload .
Combination of nitrate and hydralazine may improve prognosis in black
patients.
3-Angiotensin-converting enzyme (ACE) inhibition therapy: This
interrupts the vicious circle of neurohumoral activation. Whilst the major
benefit of ACE inhibition in heart failure is a reduction in afterload, it
also reduces preload.
ACE inhibitors can produce a substantial improvement in effort tolerance
and in mortality, and reduce the remodeling of heart.
4-Beta-adrenoceptor blocker therapy: Beta-blockade helps to counteract
the deleterious effects of enhanced sympathetic stimulation and reduces
the risk of arrhythmias and sudden death. When initiated in standard
doses, they may precipitate acute-on-chronic heart failure, but when
given in small incremental doses (e.g. bisoprolol started at a dose of 1.25
mg daily, and increased gradually over a 12-week period to a target
maintenance dose of 10 mg daily other BB used metaprolol, and
carvidolol), they can increase ejection fraction, improve symptoms,
reduce the frequency of hospitalisation and reduce mortality in patients
with chronic heart failure.
5-Digoxin : can be used to provide rate control in patients with heart
failure and atrial fibrillation. In patients with severe heart failure (NYHA
class III–IV), digoxin reduces the likelihood of hospitalization for heart
failure, although it has no effect on long-term survival.
6-Amiodarone: This is a potent anti-arrhythmic drug that has little
negative inotropic effect and may be valuable in patients with poor left
ventricular function. It is only effective in the treatment of symptomatic
arrhythmias.
Problem 30
pituitary gland
Case history
An 18-year-old Petrol attendant , was referred to your clinic with Obesity
, he had no other complaints . He achieved all of his mile – stones
normally until the age of 13 years , when he noticed that he was smaller
than his class-mates . He was of normal intelligence , but he had
gradually become slow and apathetic . On examination a very shy , obese
boy with no evidence of beard , absent pubic & axillary hair , small
external genitalias and testicles and absent rugosity of his scrotum . His
height was less than expected when compared with the size of his parents
& siblings .
Confrontation Perimetry & fundoscopy are normal with no limb
weakness , but his reflexes showed a slow relaxation phase . The breasts
were rather prominent , and a minimal serous secretion seen from left
nipple.
CPB ESR , B urea , S . Creatinine , Electrolytes , LFT all within normal
Serum Ca2+ 2. 56mmol/L Phosphate 1.15 mmol/L Urine Analysis
Normal .
1 . What is the differential diagnosis ?
2 . What are the additional investigation would you consider ?
3 . What is the final diagnosis ?
4 . How could you manage this patient ?
5 . When & how to do Insulin Tolerence test ?
6 . How is replacement testosterone therapy usually administered in this
patient ?